client login - exam 1 questions and answers · 2015-09-29 · 18. which of the following most...

56
American Society of Nuclear Cardiology 2015 Nuclear Cardiology Board Exam Preparation Course Washington, DC EXAM 1 – Questions and Answers Topics Covered: Instrumentation Physics Production of Radionuclides and Radiopharmaceuticals Part 1&2 1. A point source of 99m Tc is placed in front of a gamma camera with a parallel hole collimator. As the point source is moved away from the collimator (ie, 5 cm to 15 cm but remain within the field of view), which of the following will happen to the count rate? a. will drop by 50% b. will increase by 50% c. remains unchanged d. increases and the image remains unchanged Explanation: Normally as a source of radiation is moved away from a detector, inverse square law indicates a drop off in count rate. However in the particular case of a collimated system, the number of collimator holes that allow events through to the crystal increases as a point source is moved away from the collimator face. These 2 factors cancel each other out so that the effective count rate remains unchanged. 2. After eluting a 99m Tc generator the Sodium Pertechnetate eluate must be assayed to determine the a. Tc99m activity (mCi) b. 99 Mo activity (uCi) c. shelflife of the elution. d. all of the above Ref: 10CFR35.204 Permissible molybdenum99 concentration. 3. Spatial resolution will _________ with increasing distance from a parallel hole collimator. a. degrade b. improve c. remain unchanged d. improve initially, then degrade Explanation: This is true for ALL collimators. The best resolution is always at the surface of the collimator. 4. The most probable interaction of 99m Tc, 140 keV photons in fatty tissue is: a. photoelectric absorption b. compton scatter c. pair production d. complete transmission Explanation: Photoelectric is dominant below ~ 28 keV in soft tissue and ~48 keV for bone; Compton is the most likely; for photons whose energy is less than 1.022 MeV, pair production is impossible; complete transmission is more likely for much higher energy photons.

Upload: others

Post on 20-Mar-2020

2 views

Category:

Documents


0 download

TRANSCRIPT

Page 1: Client Login - EXAM 1 Questions and Answers · 2015-09-29 · 18. Which of the following most affects the technetium yield from a 99Mo‐99mTc generator? a. Channeling b. Radiolysis

American Society of Nuclear Cardiology 2015 Nuclear Cardiology Board Exam Preparation Course 

Washington, DC 

 EXAM 1 – Questions and Answers 

 Topics Covered: Instrumentation  Physics  Production of Radionuclides and Radiopharmaceuticals Part 1&2  1. A point source of 99mTc is placed in front of a gamma camera with a parallel hole collimator. As the point 

source is moved away from the collimator (ie, 5 cm to 15 cm but remain within the field of view), which of the following will happen to the count rate? 

 a. will drop by 50%  b. will increase by 50%  c. remains unchanged d. increases and the image remains unchanged  

 Explanation: Normally as a source of radiation is moved away from a detector, inverse square law indicates a drop off in count rate. However in the particular case of a collimated system, the number of collimator holes that allow events through to the crystal increases as a point source is moved away from the collimator face. These 2 factors cancel each other out so that the effective count rate remains unchanged. 

  2. After eluting a 99mTc generator the Sodium Pertechnetate eluate must be assayed to determine the  

a. Tc99m activity (mCi)  b. 99Mo activity (uCi)  c. shelf‐life of the elution. d. all of the above 

Ref: 10CFR35.204 Permissible molybdenum‐99 concentration.  

3. Spatial resolution will _________ with increasing distance from a parallel hole collimator.  

a. degrade b. improve  c. remain unchanged  d. improve initially, then degrade  

 Explanation: This is true for ALL collimators. The best resolution is always at the surface of the collimator. 

 4. The most probable interaction of 99mTc, 140 keV photons in fatty tissue is:  

a. photoelectric absorption b. compton scatter c. pair production d. complete transmission 

 Explanation: Photoelectric is dominant below ~ 28 keV in soft tissue and ~48 keV for bone; Compton is the most likely; for photons whose energy is less than 1.022 MeV, pair production is impossible; complete transmission is more likely for much higher energy photons.  

Page 2: Client Login - EXAM 1 Questions and Answers · 2015-09-29 · 18. Which of the following most affects the technetium yield from a 99Mo‐99mTc generator? a. Channeling b. Radiolysis

  

5. Testing the first elution of every generator for 99Mo will be performed in order to assure that 99mTc 

radiopharmaceuticals will contain no more than 0.15�i of 99Mo per 1mCi of 99mTc radiopharmaceutical at 

the time of ________.   

a. elution b. administration c. kit preparation d. expiration   Ref: Kowalsky R, Falen S (2004). Radiopharmaceuticals in Nuclear Pharmacy and Nuclear Medicine, 2nd ed. Washington, DC: American Pharmacists Association. 221‐223. 

 6. The attenuation of photons in matter is a function of: 

 a. photon energy and Z number b. linear energy transfer c. photon energy and conversion electrons d. mode of decay 

 Explanation: The probability of a specific type of interaction is based on the number of protons of the matter and energy of the photon. 

 7. If 99Mo is removed from the column during the elution process the activity is considered to be a    

a. Chemical impurity b. Radiochemical impurity c. Radionuclide impurity d. Quality control error 

 Ref: Kowalsky R, Falen S (2004). Radiopharmaceuticals in Nuclear Pharmacy and Nuclear Medicine, 2nd ed. Washington, DC: American Pharmacists Association. 221‐223.  

 8. Photons, undergoing photoelectric absorption in tissue, produce 

a. annihilation photons b. positrons c. electrons and characteristic x‐rays 

Page 3: Client Login - EXAM 1 Questions and Answers · 2015-09-29 · 18. Which of the following most affects the technetium yield from a 99Mo‐99mTc generator? a. Channeling b. Radiolysis

d. gamma photons of lower energy than the primary photons  

Ref: Chandra, Ramesh (1998). Nuclear Medicine Physics. P 61‐63  

9. Which organ would most likely receive a dose 0.03rem/uCi if 99Mo contamination from an elution was administered?  a. Liver b. Stomach c. Bladder d. Thyroid 

 Ref: D. Shearer. Radiation dose from radiopharmaceuticals contaminated with molybdenum‐99. J Nucl Med. 1988:29:695‐700. 

 10. In a conventional SPECT camera, each row of pixels is used to reconstruct which of the following:   

a. entire tomographic image set b. single transaxial slice c. single short axis slice d. single coronal slice 

 Explanation: A sinogram is a single row of pixels, repeated for each planar projection. Each sinogram is used to reconstruct a single transaxial slice.  Ref. Powsner RA, Plamer MR, Powsner ER. Essentials of nuclear medicine physics and instrumentation, Third Edition. Wiley‐Blackwell. 2013. P. 97,134‐136.  

 11. If you received a 40 mCi dose of FDG at 8:00 AM and inject a patient at noon, how much activity would 

there be at the time of injection?    

a. 40 mCi b. 20 mCi c. 10 mCi d. 5 mCi 

 Explanation: The half‐life of 18F is ~ 2 hours (110 minutes).  So, at 10AM you will have ~20 mCi, and at Noon, you will have ~10 mCi. 

 12. What is the radiochemical purity needed for most Tc99m radiopharmaceuticals?  

a. 100% b. 90% c. 80% d. 70% 

 Ref: Kowalsky R, Falen S (2004). Radiopharmaceuticals in Nuclear Pharmacy and Nuclear Medicine, 2nd ed. Washington, DC: American Pharmacists Association. 416.] 

 13. In an Anger Gamma Camera, low energy electrons are ejected when light strikes the:  

a. Anode  b. Photocathode c. Sodium iodide crystal  d. Dynode array  

 

Page 4: Client Login - EXAM 1 Questions and Answers · 2015-09-29 · 18. Which of the following most affects the technetium yield from a 99Mo‐99mTc generator? a. Channeling b. Radiolysis

Explanation: In a gamma camera, the gamma rays interact with the crystal to produce light. The light then strikes the photocathode. The photocathode is negatively charged. Low energy electrons are knocked out of the photocathode by the light photons and are accelerated towards the positively charged dynode array.  

 14. Which of the following radiotracers is produced by a cyclotron?  

a. Technetium‐99m b.    Molybdenum‐99 c.    Thallium‐201 d.    Rubidium‐82 

 Ref: Kowalsky R, Falen S (2004). Radiopharmaceuticals in Nuclear Pharmacy and Nuclear Medicine, 2nd ed. Washington, DC: American Pharmacists Association. 416. 

 15. The Mo‐99/Tc‐99m generator is an example of which theory of equilibrium?  

a. First order b. Secular c. Specific d. Transient 

 Ref: Kowalsky R, Falen S (2004). Radiopharmaceuticals in Nuclear Pharmacy and Nuclear Medicine, 2nd ed. Washington, DC: American Pharmacists Association. 427.  

16. What is the physical half‐life of 99Molybdenium:  

a. 26 days b. 66 hours     c. 6 hours d. 10 days 

 Reference: New radiotracers in cardiac imaging, principles and applications. Taillefer, Tamaki Appleton and Lange, 1999 

 17. The determining characteristic between a stable and unstable nucleus is  

a. number of protons b. proton to neutron ratio c. vacancies in the K‐shell d. number of neutrons 

 Explanation: The P:N ratio determines the balance of coulombic (repelling)  and attractive (binding) forces within the nucleus. When those forces are matched stability results. When those forces are mismatched instability results. 

 18. Which of the following most affects the technetium yield from a 99Mo‐99mTc generator?  

a. Channeling b. Radiolysis c. Decay to Tc99 d. Transient equilibrium 

 Ref: Kowalsky R, Falen S (2004). Radiopharmaceuticals in Nuclear Pharmacy and Nuclear Medicine, 2nd ed. Washington, DC: American Pharmacists Association. 427. 

 19. Sodium Iodide crystals are not ideal for imaging positron emitting radiotracer because of   

a. low light yield 

Page 5: Client Login - EXAM 1 Questions and Answers · 2015-09-29 · 18. Which of the following most affects the technetium yield from a 99Mo‐99mTc generator? a. Channeling b. Radiolysis

b. high linear attenuation coefficient c. higher expense d. low stopping power 

 Ref: Saha, Gopal (2005), Basics of PET Imaging. P.20‐21 

 20. Isomeric transitions result in emissions of which of the following?  

a. Positrons b. Alpha Particles c. Neutrons d. Characteristic Radiations 

 Explanation: In an isomeric transition only the energy state(s) of 1 (or more) nucleons changes. Positrons are given off when a proton changes into a neutron in positron decay; an alpha particle would decrease the total number of nucleons by 4 (2 neutrons and 2 protons); the emission of a neutron would decrease the number of neutrons by one; characteristic radiation is given off when an electron gives off energy as it moves to fill a vacancy created by another electron being ejected from the atom through the process of internal conversion, which is one of the 2 forms of isomeric transition. 

 21. Which of the following tracers requires an on‐site cyclotron for its production?  

a. Oxygen 15‐water b. Rubidium Chloride 82 c. Technetium Tc99m tetrofosmin d. Fluorine F18 fludeoxyglucose 

 Ref: Kowalsky R, Falen S (2004). Radiopharmaceuticals in Nuclear Pharmacy and Nuclear Medicine, 2nd ed. Washington, DC: American Pharmacists Association. 346.  

22. The collimator on a SPECT camera is changed from a low energy all purpose (LEAP) collimator to a low energy high‐resolution (LEHR) collimator. In order to acquire the same number of counts: 

 a. a shorter acquisition time can be used b. a longer acquisition time will be needed c. camera to patient distance will need to be increased d. camera to patient distance will need to be decreased 

 Explanation: The trade‐off for higher resolution collimation will be lower sensitivity.  Ref. Holly TA, Abbott BG, Al‐Mallah M, Calnon DA, Cohen MC, DiFilippo FP, Ficaro EP, Freeman MR, Hendel RC, Jain D, Leonard SM, Nichols KJ, Polk DM, Soman P; American Society of Nuclear Cardiology. Single photon‐emission computed tomography. J Nucl Cardiol. 2010 Oct;17(5):941‐73.  Ref. Powsner RA, Plamer MR, Powsner ER. Essentials of nuclear medicine physics and instrumentation, Third Edition. Wiley‐Blackwell. 2013. P. 73‐79.  

23. When compared to neutron capture production, which of the following statement is true on fission production of 99Molybdenium: 

 a. Costs less to prepare b. Has a lower concentration c. Has a higher specific activity    d. Uses more shielding  Reference: New radiotracers in cardiac imaging, principles and applications. Taillefer, Tamaki Appleton and Lange, 1999 

 

Page 6: Client Login - EXAM 1 Questions and Answers · 2015-09-29 · 18. Which of the following most affects the technetium yield from a 99Mo‐99mTc generator? a. Channeling b. Radiolysis

24. Atoms that have the same number of neutrons but differ in the number of protons are classified as the following: 

 a. Isotones b. Isotopes c. Isomers d. Isobars 

 Explanation:  Isotones have the same number of neutrons (different elements); Isotopes have the same number of protons (same elements); Isobars have the same number of nucleons (different elements); Isomers are the same nuclide with different energy (excited) states.  

25. Alumina breakthrough from a 99mTc generator is evaluated by:  

a. Electrophoresis b. Paper thin‐layer chromatography c. Gel chromatography d. Colorimetric test    

 Reference: New radiotracers in cardiac imaging, principles and applications. Taillefer, Tamaki Appleton and Lange, 1999 

 26. Which of the following types of radiation has the greatest linear energy transfer (LET)?  

a. Alpha b. Beta c. Positron d. Gamma 

 Explanation: Particles with high mass lose their energy over a short path length, resulting in a high LET.  Of the particles listed, alpha particles have the highest mass (2 protons + 2 neutrons). 

  27. Spatial resolution in PET can best be improved by which of the following?  

a. Increasing the detector ring diameter  b. Decreasing the collimator hole size c. Increasing the coincidence timing window  d. Using positrons with lower average kinetic energy 

 Explanation: Because of the non‐colinearity of the 511 keV photons emitted a larger diameter detector ring results in lower spatial resolution; there is no collimator employed in PET; increasing the CTW would increase the number of random coincidences used to form the image thereby degrading the image; with a decrease in positron kinetic energy the positron will go a shorter distance between the site of emission and where it annihilates. Since it is the site of annihilation that we determine this would improve spatial resolution. 

 28. Which of the following SPECT myocardial perfusion imaging agent has the highest first pass myocardial 

extraction fraction:  

a. 201 Thallium      b. 99mTc‐sestamibi c. 99mTc‐tetrofosmin d. 99mTc‐furifosmin 

 Reference: New radiotracers in cardiac imaging, principles and applications. Taillefer, Tamaki Appleton and Lange, 1999 

 

Page 7: Client Login - EXAM 1 Questions and Answers · 2015-09-29 · 18. Which of the following most affects the technetium yield from a 99Mo‐99mTc generator? a. Channeling b. Radiolysis

29. Which of the following PET myocardial perfusion imaging agent has the highest myocardial extraction fraction: 

 a. 82‐Rubidium b. 13N‐ammonia c. 15O‐water     d. 18F‐Flurpiridaz 

  Reference: New radiotracers in cardiac imaging, principles and applications. Taillefer, Tamaki Appleton and Lange, 1999 

 30. Which of the following would result in increased system spatial resolution for a gamma‐camera with a 

parallel‐hole collimator?  

a. Longer collimator bore  b. Larger diameter holes c. Keeping everything else constant and making the collimator septa thinner. d. Moving farther from the face of the collimator. 

 Explanation: In the absence of septal penetration, hole‐geometry determines the spatial resolution of a parallel‐hole collimator at a given distance. Longer holes are more selective in the locations they allow photons to originate from, they thus have better spatial resolution; making the collimator hole diameter larger makes the collimator less selective hence decreases spatial resolution; making the septa thinner does not change hole geometry and allows more penetration so may make resolution worse; spatial resolution gets worse with distance from the face of parallel‐hole collimators.  

31. Which of the following SPECT myocardial perfusion imaging agent has a neutral charge:  

a. 9mTc‐sestamibi b. 99mTc‐teboroxime c. 99mTc‐tetrofosmine  d. 99mTc‐furifosmine  Reference: New radiotracers in cardiac imaging, principles and applications. Taillefer, Tamaki Appleton and Lange, 1999 

 32. As compared to an analog scintillation camera, an advantage of a digital scintillation camera is higher  

Page 8: Client Login - EXAM 1 Questions and Answers · 2015-09-29 · 18. Which of the following most affects the technetium yield from a 99Mo‐99mTc generator? a. Channeling b. Radiolysis

a. Light yield b. Stopping power c. Dead time d. Energy resolution 

 Explanation:  Light yield and stopping power are properties of the crystal and have nothing to do with the electronics of the camera; higher dead time would be a disadvantage; better spatial resolution and better energy resolution are possible because of corrections can be implemented with software, enabling the use of more sophisticated algorithms..   

33. Which of the following SPECT myocardial perfusion imaging agent is labelled at room temperature:  

a. 99mTc‐sestamibi b. 99mTc‐tetrofosmin   c. 99mTc‐teboroxime d. 201‐thallium 

 Reference: New radiotracers in cardiac imaging, principles and applications. Taillefer, Tamaki Appleton and Lange, 1999 

 34. What is the main photon energy of 99Mo:  

a.   740 and 780 Kev b.    600 and 640 Kev c.   511 Kev d.   140 and 180 Kev. 

 Kowalsky R, Falen S (2004). Radiopharmaceuticals in Nuclear Pharmacy and Nuclear Medicine, 2nd ed. Washington, DC: American Pharmacists Association. 416.  Explanation: The main photon energies of 99Mo ( 740 and 780 KeV) allow to use the principle of breakthrough in order to evaluate the amount of 99Mo in a vial of 99mTc ( 140 KeV). Gamma rays from 99mTc will be stopped by a certain amount of lead shield while the gamma rays from 99Mo will "breakthrough" the lead and will be detectable.  

35. Compared to filtered back‐projection, Ordered Subset Expectation Maximization (OSEM‐iterative reconstruction) for SPECT a. Is faster to compute b. Increases ramp filter artifact c. accepts corrections for attenuation d. easier to implement 

 Explanation: Iterative reconstruction techniques are able to directly implement corrections for attenuation and scatter, which cannot be done in filtered‐backprojection at the expense of much longer computation times due to increased mathematical complexity. b is incorrect because filtered‐backprojection is an exact mathematical solution in the ideal case (no attenuation, scatter, noise, etc).  Ref. Powsner RA, Plamer MR, Powsner ER. Essentials of nuclear medicine physics and instrumentation, Third Edition. Wiley‐Blackwell. 2013. P. 149‐152. 

 36. Which of the following redistribution curves represents a myocardial segment with a 50% stenosis? 

Page 9: Client Login - EXAM 1 Questions and Answers · 2015-09-29 · 18. Which of the following most affects the technetium yield from a 99Mo‐99mTc generator? a. Channeling b. Radiolysis

 a. 1 b. 2 c. 3 

 Explanation: Curve #1 represents normal flow with high uptake and redistribution over time.  Curve #2 is an ischemic segment with 50% stenosis.  Initial flow is reduced by half initially. With redistribution it approaches normal. Curve #3 represents a mixture of scar and ischemia.  It has half the initial uptake and half of the redistribution thus resulting in a parallel curve as the normal segment. 

 37. The radiation dose is highest in which organ for a standard dose of 201Tl? 

a. Gallbladder b. Thyroid c. Kidneys d. Upper large intestine 

 Explanation:    Doses (rad/3 mCi)   Gallbladder  0.93   Thyroid        2.24   Kidneys        4.40   Upper large intestine   3.60  38. The radiation dose is highest in which organ for a standard dose of 99mTc Sestamibi? 

a. Gallbladder b. Thyroid c. Kidneys d. Upper large intestine 

 Explanation: 

  Doses (rad/30 mCi)   Gallbladder  2.44   Thyroid        0.63   Kidneys        2.01   Upper large intestine 4.77: 

1 2 3

Page 10: Client Login - EXAM 1 Questions and Answers · 2015-09-29 · 18. Which of the following most affects the technetium yield from a 99Mo‐99mTc generator? a. Channeling b. Radiolysis

 39. Initial myocardial uptake of a Tl‐201 injection is approximately  

a. 2% b. 4% c. 6% d. 8% 

 Explanation: Initial myocardial uptake of a Tl‐201 injection is 4% for thallium as opposed to 99m Tc Sestamibi and Tetrofosmin which are around 1.2%.    40. If a nucleus decays by isomeric transition which of the following might be emitted in the process? 

a. A Positron b. An Alpha Particle c. A Neutron d. Characteristic Radiation 

 Explanation: In an isomeric transition only the energy state(s) of 1 (or more) nucleons changes. Positrons are given off when a proton changes into a neutron in positron decay; an alpha particle would decrease the total number of nucleons by 4 (2 neutrons and 2 protons); the emission of a neutron would decrease the number of neutrons by one; characteristic radiation is given off when an electron gives off energy as it moves to fill a vacancy created by another electron being ejected from the atom through the process of internal conversion, which is one of the 2 forms of isomeric transition. 

 41. Which of the following types of radiation has the greatest penetration in human tissue? 

a. Alpha b. Beta c. Positron d. Gamma 

 Explanation:  Alpha radiation has the lowest penetrating power and can be stopped with a sheet of paper.  Beta and Positron radiation have medium penetrating power and can only penetrate to the “germinal layer” of skin (where new skin develops); most can be stopped with a sheet of aluminum or plastic.  Gamma radiation has the highest penetrating power of the 4 and can travel through the body; they can be stopped with dense substances such as lead and concrete.  Reference:  Health Physics Society FAQ, “What Types of Radiation are There?”, 

http://hps.org/publicinformation/ate/faqs/radiationtypes.html  42. According to the NRC, the maximum allowable Mo‐99 “ breakthrough” in 99m‐Tc eluate at time of injection is 

a. <0.15 µCi Mo/mCi Tc b. > 0.05 mCi Mo/mCi Tc c. <0.25 µCi Mo/mCi Tc d. < 0.015 µCi Mo/mCi Tc 

 Explanation: Each time a generator is elluted, impurities will be present in the eluated daughter radionuclide. For instance, in the case of 99m‐Tc eluate obtained from 99‐Mo generator, 99‐Mo impurities will be present. According to the NRC, Mo‐99 contamination in 99m‐Tc eluate must be measured following EACH elution. Presence of Mo‐99 as an impurity in the eluted Tc‐99m serves as a source of higher radiation dose and contributes to poorer image quality. Thus measurement of 99‐Mo breakthrough is an essential QC step in a Mo‐99/Tc‐99m GENERATOR.   43. Which one of the following molecular imaging techniques has shown promise for detection of early 

calcification process in the vascular wall? a. 18F‐FDG PET b. 18F‐NaF PET c. 99mTc‐tetrofosmin SPECT d. 201Thallium SPECT 

 

Page 11: Client Login - EXAM 1 Questions and Answers · 2015-09-29 · 18. Which of the following most affects the technetium yield from a 99Mo‐99mTc generator? a. Channeling b. Radiolysis

Explanation: 18F‐FDG is taken up by inflammatory cells and may detect inflammatory diseases. 18F‐NaF was initially introduced as a bone‐imaging agent. Recent data suggest that it may detect foci of early calcification in the vessel wall.  99mTc‐tetrofosmin SPECT and 201Thallium SPECT are used to assess myocardial blood flow.  44. Which of the following myocardial perfusion tracers has the shortest half‐life? 

a. N‐13 NH3 

b. Tc‐99m sestamibi 

c. Rb‐82 

d. F‐18 flurpiridaz 

 Explanation: N‐13 NH3: 10 min Tc‐99m sestamibi: 6 hours Rb‐82: 75 sec. F‐18 flurpiridaz: 120 min  

45. Which of the following radiotracers has the highest first pass radiotracer extraction? a. Oxygen‐15 water b. N‐13 ammonia c. Thallium 201 d. Rubidium‐82 

 Explanation: O‐15 water has the best first pass radiotracer extraction which is close to 100%. By comparison all other perfusion radiotracers have lower extraction fractions. 

   

 References: Salerno and Beller. Circulation: Cardiovascular Imaging. 2009; 2: 412‐424.  

Page 12: Client Login - EXAM 1 Questions and Answers · 2015-09-29 · 18. Which of the following most affects the technetium yield from a 99Mo‐99mTc generator? a. Channeling b. Radiolysis

American Society of Nuclear Cardiology 2015 Nuclear Cardiology Board Exam Preparation Course 

Washington, DC  

EXAM 2 – Questions and Answers   

Topics Covered:   Stress Testing, Image Interpretation and Protocols Risk Stratification Technical Aspects of Acquisition and Processing  

 1. 70 year old man with hypertension with typical angina symptoms. A rest stress technetium 99m sestamibi 

study is performed. Based on the images, the most likely finding is:   

a. Global ischemia b. TID c. Incorrect normalization d. Attenuation artifact 

  Explanation: The images reveal lack of normalization of the stress images.  The images should be normalized to the hottest pixel to allow adequate comparison between rest and stress images.  

 2. Selective use of angiography after SPECT imaging as compared to direct angiography in patients with stable 

angina results in   

a. Worse outcomes  b. Improved Outcomes c. Lower cost d. Higher cost   

Explanation: The END study showed that the cost of care was significantly reduced in those individuals who underwent cath only based upon the results of an MPI study.  Those that underwent direct catheterization had increased costs and the same mortality.  

   

Page 13: Client Login - EXAM 1 Questions and Answers · 2015-09-29 · 18. Which of the following most affects the technetium yield from a 99Mo‐99mTc generator? a. Channeling b. Radiolysis

3. Which of the following filters is ALWAYS employed in the SPECT reconstruction using filtered back‐projection? 

 a. Ramp filter  b. 9‐point smoothing filter    c. Hann filter  d. Butterworth filter  

 Explanation: The ramp filter is an integral part of the reconstruction process. The other filters are all smoothing filters and their use is optional.  4. Which of the following is a common reason for low specificity of SPECT MPI in relation to coronary 

angiography?   

a. Reclassification bias b. Referral bias c. High positive predictive value  d. Prevalence bias   

Explanation: This  is a classic example of  referral bias. Referral bias  (or verification bias) occurs because patients with  normal  SPECT MPI  rarely  go  to  coronary  angiography.  Thus,  in  retrospective  analyses  of  the  diagnostic accuracy of SPECT MPI using angiographic CAD as the standard, the study cohorts have few patients with normal angiography and SPECT. This leads to referral bias and results in a spurious decreases in specificity and a spurious increase in sensitivity. 

 5. A 49 year old man with history of hypertension and tobacco use presents to the emergency room with 6 

week history of intermittent chest pain. His baseline EKG shows LVH with repolarization changes.  His cardiac enzymes at arrival and 6 hours later are negative.  Which is the most appropriate next step in his care? 

 a. cardiac catheterization b. exercise stress testing c. exercise MPI d. echocardiogram 

 Explanation: The answer could be either b or c.  This patient does not have ST elevation and with negative enzymes, there is no urgent indication to go immediately to the cath lab.  The degree of repolarization abnormalities is not defined.  If there is more than 1 mm of ST segment depression at baseline then the patient should undergo stress testing with imaging.  In the exercise stress testing guidelines it is a class 1 indication to do exercise only stress testing to evaluate patient with recurrent symptoms suggesting ischemia after revascularization.  In the more recent Stable Ischemic Guidelines, it is a class IIa indication to do imaging stress test in patient with new or worsening symptoms not consistent with unstable angina and who have at least moderate physical functioning and no disability comorbidity, previously requiring imaging with exercise or known multivessel disease.  Imaging with exercise would be considered appropriate in this patient according to the most recent appropriateness use criteria.   2012 ACCF/AHA/ACP/AATS/PCNA/SCAI/STS Guideline for the Diagnosis and Management of Patients With Stable Ischemic Heart Disease. Circulation. 2012; 126: e354‐e471  ACC/AHA PRACTICE GUIDELINES ACC/AHA Guidelines for Exercise Testing A Report of the American College of Cardiology/American Heart Association Task Force on Practice Guidelines (Committee on Exercise Testing) JACC Vol. 30, No. 1 July 1997:260–315   

ACCF/AHA/ASE/ASNC/HFSA/HRS/SCAI/SCCT/SCMR/STS 2013 multimodality appropriate use criteria for the detection and risk assessment of stable ischemic heart disease. 

Wolk MJ, Bailey SR, Doherty JU, Douglas PS, Hendel RC, Kramer CM, Min JK, Patel MR, Rosenbaum L, Shaw LJ, Stainback RF, Allen JM; American College of Cardiology Foundation Appropriate Use Criteria Task Force. J Am Coll Cardiol. 2014 Feb 4;63(4):380‐406. 

Page 14: Client Login - EXAM 1 Questions and Answers · 2015-09-29 · 18. Which of the following most affects the technetium yield from a 99Mo‐99mTc generator? a. Channeling b. Radiolysis

  

6. A MPI using a 99mTC agent is acquired with the PHA calibrated for 201Tl. Which of the following statements is correct regarding the image data? 

   a. Tomographic images should be reconstructed using the filter settings for 201Tl. b. The raw (unprocessed) images should be evaluated with the final report indicating the study was 

suboptimal.  c. The acquisition will need to be repeated using the correct PHA setting.  d. The raw data should be corrected using a 201Tl uniformity correction map, prior to reconstruction.   

 Explanation: The data acquired will have incorrect energy information, therefore cannot be processed.  (Unless the data was acquired in list mode, which is very uncommon.)  

 7. A 50 year old male with a history of hypertension presents with chest discomfort on and off for the past 2 

days. His physical exam is normal and initial electrocardiogram  shows left ventricular hypertrophy but no 

acute ischemic changes . Admission and 3 hour cardiac enzymes are normal. He is scheduled for stress first 

exercise Tc 99m SPECT MPI. Just when he is brought to the stress lab he develops moderate intensity 

retrosternal pain similar to his initial chest discomfort.  His vital signs are stable and repeat 

electrocardiogram is unchanged. His pain lasts 10 minutes and is almost subsided yet he is uncomfortable. 

The next most appropriate step in his diagnostic evaluation is:   

 

a. Proceed with stress testing 

b. Cardiac catheterization 

c. Inject the radioisotope 

d. Switch to pharmacologic stress 

e. Chest CT scan 

Learning objective:  Understand the advantages and disadvantages of acute chest pain scans in the emergency 

room. 

Discussion :  Injection of the radioisotope during or immediately after chest pain resolution is do evaluation of 

suspected ACS is called acute chest pain imaging. A normal chest pain scan has very high negative predictive value 

of ACS ( 99%) and patients can be safely discharged home with subsequent follow‐up . However as perfusion 

defects tend to resolve in the ensuing hours after chest pain resolves the maximum yield of chest pain imaging is 

during or shortly after resolution of chest pain. As the sensitivity decreases substantially after 2 hours of pain 

resolution chest pain imaging is not recommended beyond this window. Furthermore if patients have prior MI, new 

defects cannot be differentiated from old defects unless occurring in a different distribution which cannot be 

predicted prior to scan always. A negative chest pain scan done in the right time window as mentioned above has 

been shown to have excellent negative predictive value for acute ischemic event and favorable short term prognosis 

it does not exclude obstructive CAD. So such patients should be brought back after discharge for formal stress 

perfusion imaging to exclude CAD. As patient continues to have rest symptoms stress testing is not advisable and 

switching mode of stress is not indicated either. In absence of dynamic ST changes or positive cardiac enzymes 

cardiac cath is no indicated at this juncture and symptomatology favors ACS rather than dissection and thus CT 

scan is not the right choice.  

Reference : Kontos MC et al  Cardiol Clin. 2005 Nov;23(4):517‐30  ERASE :  Udelson JE et al ; JAMA 2002: 288:21; 2693‐2700  8. The specificity of SPECT MPI is increased by:  

a. Substitution of thallium‐201 for Tc‐99m b. Stress –only  imaging c. Attenuation correction 

Page 15: Client Login - EXAM 1 Questions and Answers · 2015-09-29 · 18. Which of the following most affects the technetium yield from a 99Mo‐99mTc generator? a. Channeling b. Radiolysis

d. The use of flow tracers with high extraction fraction    Explanation:  False positive studies due to attenuation artifacts reduce specificity. Thus, attenuation correction improves it. Stress –only imaging is useful to reduce the protocol duration and radiation dose, but has no effect on diagnostic sensitivity or specificity. Tl‐201 has a lower photon energy than Tc‐99m, and is therefore, more prone to attenuation artifacts. The use of tracers with high extraction fraction facilitates the detection of mild CAD, and may improve sensitivity.  

 9. A 65–year‐old woman is being evaluated prior to surgery for elective hip replacement. Her history is 

significant for DM, HTN and atypical chest pain. She undergoes SPECT imaging (see image).   Which is the next most appropriate step in her care? 

 a.  Cardiac catheterization b.  Proceed with surgery c.  Refer for further imaging 

 In evaluating any study, it is important to evaluate all available information such as extracardiac tracer uptake. This patient has a large amount of tracer uptake in a right breast mass.  This is elective surgery, so she should undergo workup for her breast cancer.   10. A 54‐year‐old woman with dyspnea on exertion is referred for stress testing. She undergoes stress only 

imaging.  Her history is significant for HTN. She exercise for 5 min 20 seconds of a Bruce protocol and achieves heart rate of 131 (79% MPHR).  The exam is terminated because of shortness of breath.  There are 1 mm ST depressions in leads II, III and aVF. Which is the most appropriate next step in her care? 

 a. Obtain rest study b. Cardiac catheterization c. Echocardiogram d. Medical management 

 Explanation: If abnormalities in perfusion are seen in a stress only study, then a rest study should be performed.  This patient has a large pericardial effusion likely the cause of her symptoms of dyspnea on exertion. She should undergo an echocardiogram to evaluate the effusion.  

 11. Which of the following parameters indicates a lowest risk of hard events in a patient with a normal scan? 

a. Diabetes Mellitis b. Duke treadmill score of ‐5  c.  Functional capacity  of 10 METS  d. Prior PCI 

 Explanation: Diabetics, elderly patients and those with poor functional capacity (and therefore undergoing pharmacological stress testing) are known to have a >1% annual hard event rate despite a normal stress test. It must be noted however, that a normal stress test in these groups still identifies patients at relatively lower risk compared to patients with an abnormal stress test. (Navare s, et al. J Nucl Cardiol 2004; 11:543, Hachamovitch R, et al. Circulation 2009; 120: 2197, Shaw L and Iskandrian A. J Nucl Cardiol 2004; 11: 171) 

 12. Which of the following should be used for interpretation of SPECT images? 

 a. Linear scale 

b. Sigmoidal scale 

c. Logarithmic scale 

Explanation: Linear scales should always be used when interpreting SPECT.  The gradual changes in uptake are seen better than when using sigmoidal (nonlinear) scales.  These may miss small defects. Grey scale is generally preferred but is depending on the comfort level of the reader.   Logarithmic scales may be used in regions with lower count density such as the RV but not to evaluate regional differences in LV tracer uptake.  

Page 16: Client Login - EXAM 1 Questions and Answers · 2015-09-29 · 18. Which of the following most affects the technetium yield from a 99Mo‐99mTc generator? a. Channeling b. Radiolysis

 Reference: Holly TA et al. Single photon‐emission computed tomography: http://www.asnc.org/imageuploads/ImagingGuidelineSPECTJune2010.pdf  

 13. Which of the following is a contraindication for vasodilator pharmacologic stress testing in a patient with 

history of CAD, sick sinus syndrome s/p dual chamber pacemaker placement?  

a. Mean aortic gradient of 28 mmHg 

b. Acute MI 4 days prior 

c. Systolic blood pressure < 90 mmHg 

d. High degree AV block  

Explanation:  Persistent hypotension with a systolic blood pressure <9 0 mmHg is a contraindication for vasodilator stress testing. Additional contraindications include bronchospasm, critical aortic stenosis, unstable angina, recent MI <2 days ago, high‐degree AV block without permanent pacemaker, or uncontrolled arrhythmias.   14. How many minutes after the start of dipyridamole infusion is the radiotracer injected? 

 a.   2 minutes b.   5 minutes c.   8 minutes d.   11 minutes 

 Explanation:   Maximum vasodilation of dipyridamole is 287 +/‐ 101 seconds and thus, the radiotracer should not be injected until 3‐5 minutes after the dipyridamole infusion is completed. Dipyridamole injection occurs over 4 minutes. Thus radiotracer should be injected 7‐9 minutes after the start of the dipyridamole infusion.    15. How does increasing the cutoff frequency in SPECT image reconstruction impact image resolution?  

a. No change b. Improves c. Degrades 

 Explanation: Increasing the cutoff frequency increases the resolution of images; decreasing the cutoff frequency smooths (or blurs) the images, which decreases the resolution.  16. Based on observational data, in patients with stable coronary artery disease, which of the following 

characteristics are likely to have better outcomes with coronary revascularization as compared to those receiving medical therapy? 

 a. Extensive myocardial ischemia irrespective of scar b. Extensive myocardial ischemia without large areas of scar c. Low EF without ischemia or scar d. Large area of scar and low EF 

 Explanation: Recent data from a large observational study suggests that patient with ischemia occupying > 10‐15% of the LV myocardium have better outcomes with revascularization compared to medical therapy if the ischemic burden is > 10%, or scar <10%  (Hachamovitch R, et al. Eur Heart J 2011: 32: 1012) 

  

Page 17: Client Login - EXAM 1 Questions and Answers · 2015-09-29 · 18. Which of the following most affects the technetium yield from a 99Mo‐99mTc generator? a. Channeling b. Radiolysis

  From Hachamovitch R, et al. Eur Heart J 2011; 32: 1012 

 17. For a single day rest and stress Tc‐99m sestamibi SPECT (rest followed by stress or stress followed by rest on 

a solid state CTT camera) the ratio of radiotracer injected is?   

a.  1:1 b.  1:2 c   1:3 d   1:5  

Explanation:  To best optimize the balance between radiation dose, time between rest and stress scans, a ratio of 1:3 provides adequate imaging while keeping the radiation dose as low as possible.  Normal rest doses are 8‐12 mCi and stress doses are 24‐36 mCi. This is irregardless of the type of SPECT camera used. 

 Reference: Holly TA et al. ; Single photon‐emission computed tomography: http://www.asnc.org/imageuploads/ImagingGuidelineSPECTJune2010.pdf   18. How are the sensitivity and specificity of MPI affected by post‐test referral bias?  

a. Increased sensitivity, increased specificity b. Increased sensitivity, decreased specificity c. Decreased sensitivity, increased specificity d. Decreased sensitivity, decreased specificity 

 Post‐ test referral bias or verification bias  occurs when coronary angiography is used as the reference standard to adjudicated MPI (to determine diagnostic sensitivity and specificity), and  referral to coronary angiography is primarily dictated by the results of the MPI. Thus the cohort for testing consists largely of patients with abnormal scans who have been referred for  coronary angiography. There are few patients with normal scans, who generally do not undergo angiography. Post –test referral bias is known to result in spuriously low test specificity and high sensitivity. 

Page 18: Client Login - EXAM 1 Questions and Answers · 2015-09-29 · 18. Which of the following most affects the technetium yield from a 99Mo‐99mTc generator? a. Channeling b. Radiolysis

 Ref: Rozanski A, et al. The declining specificity of radionuclide ventriculography. NEJM 1983; 309: 518  19. Leaving all other parameters the same and decreasing the number of frames from 16 to 8 in a gated MPI 

study will do which of the following:  

a. increase the temporal resolution (13)   b. decrease  spatial resolution (24)   c. increase the counts per frame (46)   d. decrease image contrast  (16)   

 Explanation: Increasing the number of frames over a set time period will increase the temporal resolution.  Since the set number of counts are split into twice as many frames, there is a decrease in the counts per frame by 50%.   20. A 59 year old diabetic women with a history of atypical chest pain presents for evaluation. She undergoes 

exercise SPECT MPI. She exercises for 4 min 30 seconds and achieves 86% PMHR.  EKG shows nonspecific ST changes.  Images are shown below.  Gated SPECT shows normal wall motion and a calculated ejection fraction of 61%.  What is the most appropriate next step in her care? 

 

Stress

Stress

Rest

Rest

Stress

Rest

  

 a.  Coronary angiography b.  Coronary CTA c.  PET imaging d.  Medical therapy 

 Explanation: This patient has a fixed anterior defect with normal wall motion consistent with breast attenuation.  If she had an associated wall motion abnormality with the perfusion defect, this would indicate infarct in the LAD territory. She should receive optimal medical therapy.  No further imaging is required.  

 21. Adjunctive exercise with pharmacologic stress testing results in which of the following?  

a. Increased liver uptake b. Increased patient motion c. Increased time of acquisition  d. Decreased symptoms 

Page 19: Client Login - EXAM 1 Questions and Answers · 2015-09-29 · 18. Which of the following most affects the technetium yield from a 99Mo‐99mTc generator? a. Channeling b. Radiolysis

 Explanation: During exercise, blood flow is diverted to skeletal muscles and away from the GI tract and therefore improves contrast.  Adjunctive exercise decreases the number and intensity of side effects, and allows for earlier acquisition. The time for acquisition does not change.   22. What instrumentation problem causes similar artifact as patient motion on SPECT myocardial perfusion 

imaging?  

a. Flood non‐uniformity  b. Ring artifact c. Energy off peak d. Incorrect center of rotation 

 Explanation: With a COR error, data from different angles of the SPECT acquisition are back‐projected to different loci of the volumetric matrix, resulting in image misregistration and artifact. With patient motion, because of differences in patient position at different times of the SPECT acquisition, a similar erroneous back projection and image misregistration occurs. 

 23. Which of the following gated SPECT imaging results indicates an increased risk for subsequent cardiac 

death?  

a. Transient ischemic dilation b. Increased Tc‐99m tetrofosmin lung activity c. Reversible perfusion defect d. An ejection fraction of 34% 

 Explanation: Low ejection fraction and large fixed defects implying a reduced ejection fraction are predictors of myocardial death. Ischemic findings are predictive of subsequent MI.   

24. Which of the factors is responsible for artifact in the images below?

 

a. Time per frame b. Motion artifact c. Breast position d. Center of rotation (COR) artifact    

Explanation:  Positioning of the breast tissue is not consistent in the rest and stress images.  Motion artifact and COR artifacts look the same.   

 

Page 20: Client Login - EXAM 1 Questions and Answers · 2015-09-29 · 18. Which of the following most affects the technetium yield from a 99Mo‐99mTc generator? a. Channeling b. Radiolysis

 25. How does decreasing the cutoff frequency in SPECT image reconstruction impact image quality?  

a. Increases the noise b. Decreases the noise c. Increases count rate  d. Decreases count rate    

 Explanation: Lowering the cutoff frequency smooths the images, therefore decreasing the resolution and the noise. There is no effect on the count rate. 

 26. Which of the following regions, by convention, is attributed to the right coronary artery?  

a. Apex  b. Basal inferolateral  c. Apical lateral  d. Basal inferoseptal   Explanation: According to the 17th segment model, the basal inferoseptal region is attributed to the RCA. (Imaging guidelines asnc.org) 

 

27. A 74‐year‐old patient had an acute myocardial infarction 3 days ago and underwent PCI of the RCA with excellent angiographic results. The patient also has moderate stenosis in the mid LAD. The patient just had adenosine SPECT dual isotope MPI and the results appear below. Based on these images, which of the following would be the best clinical approach? 

 a. Perform PCI of the LAD stenosis b. Return to the cath lab for repeat PCI of the RCA c. Intensify medical therapy d. Refer for PET imaging for viability  

Page 21: Client Login - EXAM 1 Questions and Answers · 2015-09-29 · 18. Which of the following most affects the technetium yield from a 99Mo‐99mTc generator? a. Channeling b. Radiolysis

  Explanation: SPECT reveals a predominately fixed defect in the inferior wall also involving the apical portion of the lateral wall. A small amount of peri‐infarction ischemia is present. The presence of predominate scar is against option b.  RCA is open, so the question of viability is irrelevant.  There is no evidence for hypoperfusion in the LAD distribution, so option a is incorrect. Medical therapy is most appropriate. 

 

28. Which of the following radiotracer can be used to image ischemic memory? 

a. mIBG 

b. BMIPP 

c. Tc‐99m sestamibi  

d. Rubidium‐82 

 Explanation: BMIPP can be used to image ischemic memory. I‐123 BMIPP is methylated, branched chain fatty acid. Abnormal uptake indicates a switch in myocardial metabolism from fatty acid to glucose during ischemia. Tc‐99m sestamibi and Rubidium‐82  are myocardial perfusion tracers and I‐123 mIBG is used to image myocardial sympathetic nerve terminal function.   29. Which of the following is a recommended acquisition parameter for thallium‐201 stress SPECT study?  

a. 32x32 image matrix b. 32 projections over 180 degrees c. 30% energy window at 140 keV  d. Pixel size 3.2 + 0.4 mm  

 Explanation:  Tc‐99m labeled myocardial perfusion tracers should be acquired using 60‐64 steps over 180 degrees to prevent loss of resolution. Tl‐201, due to lower count rates and increased scatter, will result in lower resolution.  Therefore, 32 projection is acceptable.  All MPI images should be acquired using at least a 64x64 matrix.  The suggested pixel size is 6.4+ 0.4 mm.  The primary energy window for Tl‐201 is 70 keV with secondary windows at 135 and/or 167 keV. 

 30. A 62 year old man with a long history of diabetes and a two month history of chest pain undergoes one day 

rest stress Tc99m SPECT MPI with attenuation correction. Based on the images, subsequent cardiac catheterization is most likely to show which of the following? 

 

Page 22: Client Login - EXAM 1 Questions and Answers · 2015-09-29 · 18. Which of the following most affects the technetium yield from a 99Mo‐99mTc generator? a. Channeling b. Radiolysis

  

a) Proximal LAD stenosis 

b) Mid LAD stenosis 

c) Distal LAD stenosis 

d) LAD and PDA stenosis 

The perfusion defect is in the distal anterior wall and apex most consistent with mid to distal LAD lesion.  LAD territory includes anterior wall and apex.  By convention the distal inferior wall is attributed to the RCA but in some individuals the LAD territory extends around the apex and supplies the distal inferior wall.  

  Reference:  Holly, TA, Abbott BG, Al‐Mallah et al. Single photon‐emission computed tomography. J Nucl Cardiol 2010;17:941–73.  31. Which of the following is true of stress‐only imaging?  

a. Approximately an additional 5 % reduction in radiation exposure compared to rest‐stress 

Page 23: Client Login - EXAM 1 Questions and Answers · 2015-09-29 · 18. Which of the following most affects the technetium yield from a 99Mo‐99mTc generator? a. Channeling b. Radiolysis

b. Transient ischemic dilation offers additive predictive value over normal stress images c. Either attenuation correction or prone imaging is integral to correct interpretation of perfusion 

defects d. Can be done only with exercise stress 

 Explanation:  Performing stress only imaging adds marked advantage with substantial radiation reduction in the range of 30‐60% over traditional rest stress imaging. TID is not assessed with stress only images. In order to conclusively assess perfusion defects as artifacts versus pathology AC or prone imaging along with gating is essential. Although exercise stress is a preferred method stress only imaging can be performed with vasodilator stress also.  32. 61‐year‐old male with a history  of hypertension, hyperlipidemia  on lisinopril and atorvastatin and 40 pack 

year history of smoking presented with chest pressure and arm pain while working in the assembly line. His clinical exam including cardiopulmonary exam was normal. His resting electrocardiogram shows sinus rhythm with non specific STT abnormality  He was referred for exercise MPI. He exercised 6 minutes on Bruce protocol without limitations, achieved 75% PMHR and had no chest pain or other hemodynamic abnormalities. His peak rest and stress electrocardiogram and SPECT images are shown.   

Resting EKG 

Peak stress EKG 

Page 24: Client Login - EXAM 1 Questions and Answers · 2015-09-29 · 18. Which of the following most affects the technetium yield from a 99Mo‐99mTc generator? a. Channeling b. Radiolysis

What is the most appropriate next step? 

 a. Repeat scan with pharmacologic stress b. Continue medical therapy c. Proceed with coronary angiography d. Perform a coronary CT scan 

 Explanation: The SPECT study overall does not show any obvious ischemia yet the Stress EKG is grossly abnormal at submaximal stress and is associated with high risk features such as ST elevation in a VR nd V1. Thus proceeding to angiography is the correct next step. Although medical therapy is indicated it is insufficient to reduce risk in this high risk test. There is no indication to repeat or do another test and images are correctly normalized.  33. The primary advantage of prone imaging is decreased 

a. anteroseptal attenuation  b. inferior wall attenuation    c. imaging time  d. Scatter from liver activity  

 Explanation: Prone imaging has been reported to produce less inferior wall diaphragmatic attenuation than supine imaging.  This is done in addition to supine imaging because prone imaging may cause an increased artifactual anteroseptal defect, therefore adding time to the procedure.  Prone imaging has no affect on liver activity.  34. A 65 year old man with exertional shortness of breath presents for stress testing. His past medical history is 

significant for hypertension and, hyperlipidemia.  His current medications include lisinopril 20 mg, HCTZ 25 mg and, atorvastatin 40 mg . His vitals are BP 142/82, HR 82 bpm, BMI 26.7 and exam is benign.  EKG shows LBBB. Which of the following is the most appropriate stress test?  

a. One day low dose stress, high dose regadenoson stress MPI b. Two day high dose rest, high dose stress exercise MPI  c. Two day high dose rest, high dose stress regadenosone MPI  d. One day low dose stress, high dose stress exercise MPI  e. One day low dose stress, high dose dobumtamine 

 

Page 25: Client Login - EXAM 1 Questions and Answers · 2015-09-29 · 18. Which of the following most affects the technetium yield from a 99Mo‐99mTc generator? a. Channeling b. Radiolysis

Explanation: Because  of his underlying LBBB he should undergo pharmacologic stress testing and not exercise stress testing to avoid potential septal defects.  Dobutamine would cause the same potential false positive septal defect. In an effort to minimize radiation exposure, low dose rest and high dose stress is the most apporpriate answer.   35. A 59 year old man with exertional chest pain while jogging presents for stress testing. His past medical 

history is significant for diabetes, hypertension, hyperlipidemia and obesity.  His medications include metformin, losartan, aspirin and simvastatin.  His exam is significant for blood pressure of 148/90, HR 76 bpm, BMI 39, normal cardiovascular exam.  EKG shows non‐specific ST segment changes.  Which of the following is the most appropriate stress test?  a. One day low dose rest, high dose regadenoson stress MPI b. Two day high dose stress, high dose rest exercise MPI c. Two day high dose rest, high dose stress regadenoson MPI d. One day low dose stress, high dose exercise MPI e. One day low dose stress, high dose dobutamine MPI 

He has no limitations for exercise therefore should undergo exercise stress testing. Because of baseline ST  T wave changes imaging is appropriate.  Because of his increased BMI he should undergo two day high dose testing. If his initial high dose stress is normal he does not need to undergo rest imaging.   36. A 48 year old man with typical exertional symptoms undergoes exercise stress testing with MPI.  He 

develops chest discomfort during 3rd stage of Bruce protocol at a heart rate of 125 bpm (73% predicted max).  His EKG shows 2.0 mm horizontal ST depressions in leads II, III and aVF.  Which of the following is the next best step? 

 a. Inject radiotracer  b. Repeat stress testing with pharmacologic agent c.  Urgent cardiac catheterization d. Exercise stress echocardiogram 

 Even though this patient has not achieved 85% of predicted maximum during his stress test, he is symptomatic with 

EKG changes. Injection at this time will help localize and risk stratify him. If he had ST elevations that urgent cath 

should be performed.  

37. A patient undergoing pharmacologic stress SPECT develops grand mal seizures shortly after injection of regadenoson. The next best treatment option for this adverse event is intravenous 

a. metoprolol b. cardizem c. aminophylline d. diazepam e. theophylline 

 Explanation: Seizures have been documented as a side effect of regadenoson in post markettng  studies. This is likely due to A2a receptor stimulation  of regadenoson on brain receptors in hippocampal and other areas  lowering seizure threshold. Thus  although aminophylline is commonly used for reversal of  regadenoson side effects, in this case it would be contraindicated as it also lowers seizure threshold and may compound effects of regadensoon. Intravenous benzodiazepenes would be best option of the choices given.    38. A 77 year old man with hypertension, diabetes, spinal stenosis and COPD presents with worsening angina.  

His current medications include simvastatin, metoprolol succinate, aspirin and metformin.  His exam is significant for BP 144/72. HR 72 distant heart sounds, clear lungs. He is scheduled for stress testing.  Which of the following is the most approppriate stress agent? 

a. Adenosine b. Exercise c. Dipyridamole d. Regadenoson 

Page 26: Client Login - EXAM 1 Questions and Answers · 2015-09-29 · 18. Which of the following most affects the technetium yield from a 99Mo‐99mTc generator? a. Channeling b. Radiolysis

 

Explanation:  Because of his spinal stenosis he likely will not be able to achieve 85% of predicted heart rate and 

therefore pharmacologic stress testing is appropriate.  Regadenoson has the shortest half life of the 

pharmacologic agents and less cross reactivity with  A2b receptors (bronchospasm). He has no active wheezing 

on exam and therefore can undergo regadenoson stress testing.  

39. Which of the following findings indicates the highest risk of adverse cardiovascular events?  

a. TID with normal perfusion 

b. A small inferior reversible perfusion defect in a patients able to exercise for 12 METS without angina 

c. Resting LV ejection fraction of 25% in a patient with congestive heart failure 

d. Non‐sustained ventricular tachycardia during exercise without angina or ECG changes 

Explanation: Severe LV dysfunction is a predictor of adverse risk, particularly in patients with heart failure symptoms. TID with normal perfusion is often associated with normal coronaries, and its association with an adverse prognosis is being increasingly questioned in recent studies. A small perfusion defect in asymptomatic patients with good functional capacity, and non‐sustained VT during exercise, are not associated with adverse risk. 

 40. C ompared to SPECT myocardial perfusion imaging, CT coronary angiography 

 a. has a higher sensitivity to diagnose left main disease 

b. is more appropriate in patients with renal failure 

c. has better specificity in patients with calcified coronaries  

d. is less affected by atrial fibrillation 

Explanation: The typical performance characteristic of CTCA is a high negative predictive value and a modest positive predictive value. Proximal coronary arteries are well visualized and the sensitivity for proximal disease is high.  It cannot be used in patients with renal failure, fast heart rates unresponsive to beta blockers, atrial fibrillation, and patients unable to hold their breath. (Marwick TH, et al. JACC 2015; 65: 2747‐56) 

41. Strategies to identify attenuation artifacts include the following except: 

 a. supine and prone imaging b. upright and supine imaging c. gated imaging d. CZT‐crystal detectors 

 Explanation: Cameras based on the solid state (CZT crystal) technology offer several advantages including greater energy resolution and high sensitivity (primarily based on collimator configuration). They do not reduce attenuation artifacts.  42. Which of the following is an unlikely cause of false‐positive myocardial perfusion scans? 

 a. LBBB b. Obesity c. caffeine intake d. arms down imaging 

 Explanation: caffeine is a competitive antagonist of the adenosine receptor and can produce false negative pharm stress myocardial perfusion scans. All the other listed choices produce false positive perfusion scans.  43. A 56 year old man underwent exercise Tc‐99m SPECT imaging, and was found to have a defect in the LAD 

territory which occupied 8% of the LV myocardium. He was started on aspirin, beta blockers and a statin with good relief of his mild exertional angina. A year later, he presents with exertional dyspnea. Which of the following is true? 

Page 27: Client Login - EXAM 1 Questions and Answers · 2015-09-29 · 18. Which of the following most affects the technetium yield from a 99Mo‐99mTc generator? a. Channeling b. Radiolysis

 a. This is an inappropriate use of SPECT b. Pharmacologic stress should be performed c. Beta blockers should not be stopped prior to testing d. CT coronary angiography is indicated 

 Explanation: Beta blockers should not be stopped when stress testing is performed for the assessment of symptoms in patients with known CAD. They do reduce diagnostic sensitivity and therefore, should be stopped 24 hours prior to diagnostic testing in patients with suspected CAD. For patients with a prior stress test, repeating the test for new symptoms is considered a “Maybe appropriate” indication. There is no indication to use pharm stress or coronary angiography preferentially in this situation.  44. Which of the following is true about the evaluation of CAD in patients with diabetes mellitus? 

 a. CTCA is inappropriate due to a high prevalence of heavily calcified coronary b. Asymptomatic diabetics have a high prevalence of myocardial perfusion abnormalities and should be 

routinely screened c. The presence of normal myocardial flow reserve on PET is a powerful indicator of a benign prognosis d. Pharmacological stress is preferred over exercise stress 

 Explanation: The concept of coronary equivalency of DM was suggested by a cross sectional study showing that incident MI was similar between patients with DM without known CAD, and patients with known CAD. Subsequent studies have questioned this concept, particularly the finding of no or minimal coronary calcium in approximately 30% of patients with DM. Contrary to prior observational studies , the prospective DIAD  study showed a low prevalence of myocardial perfusion defects in asymptomatic diabetic patients (6% prevalence of significant ischemia). The presence of a normal myocardial flow reserve is indicative of relatively benign (<1% cardiac mortality) in the intermediate (2‐3 year) term 

45. Which of the following is the most plausible explanation for transient cavity dilation of the left ventricle on 

Tc‐99m SPECT imaging in subjects with significant multi‐vessel ischemia?  

a. True cavity dilation of the left ventricle post exercise 

b. Ischemic regional left ventricular systolic dysfunction  

c. Global subendocardial hypo‐perfusion from ischemia 

d. Ischemic regional left ventricular diastolic dysfunction 

Explanation: A is incorrect as true cavity dilation typically resolves in a few minutes after termination of exercise and does not last until SPECT imaging performed 15 30 minutes later. B is not correct as it results in reversible regional wall motion abnormalities. C is correct and likely the most plausible explanation for TID. D is incorrect as diastolic dysfunction does not result in TID   Reference: 1 Holly et al. Single photon‐emission computed tomography. Journal of Nuclear Cardiology  October 2010, Volume 17, Issue 5, pp 941‐973 

46. A 45 year old man with chest pain undergoes a CT coronary angiogram study which demonstrated an anomalous right coronary artery from the left aortic sinus with an interarterial course. The best mode of stress to image the hemodynamic significance is  

a. Adenosine infusion b. Handgrip exercise c.  Exercise treadmill stress d. Dypyridamole infusion 

 Explanation: C is correct. Exercise stress is the preferred mode of stress to identify hemodynamic significance of an anomalous coronary artery A and D are incorrect, as vasodilator stress provokes differential hyperemia and not true myocardial ischemia B is incorrect as handgrip exercise may be inadequate to provoke myocardial ischemia. 

Page 28: Client Login - EXAM 1 Questions and Answers · 2015-09-29 · 18. Which of the following most affects the technetium yield from a 99Mo‐99mTc generator? a. Channeling b. Radiolysis

American Society of Nuclear Cardiology 2015 Nuclear Cardiology Board Exam Preparation Course 

Washington, DC  

EXAM 3 – Questions and Answers   

Topics Covered: 

Gated Perf SPECT  

Radionuclide Ventricular Function Imaging: Basic Lab Techniques  

Radionuclide Ventricular Function Imaging: Interpretation and Clinical Applications  

Artifact Recognition   1. The left ventricular endocardial border of a myocardial perfusion SPECT scan may be sharpened using which 

one of the following acquisition/processing modifications 

a. Substitution of thallium‐201 for either Tc‐99m sestamibi or Tc‐99m tetrofosmin     b. Substitution of a LEAP collimator for a LEHR collimator   c. Substitution of a 0.62 critical frequency for a 0.40 critical frequency while applying the Butterworth 

filter  d. Increasing the detector‐to‐heart radius of rotation  

 Explanation: Spatial resolution is lower with thallium‐201 vs. Tc‐99m, a LEAP collimator vs. a LEHR collimator, and an increase in detector‐to‐heart distance. Increasing the critical frequency increases resolution (at the expense of increasing noise).  2. A 55 year old female underwent Tc‐99m tetrofosmin myocardial perfusion SPECT for suspected coronary 

artery disease. The patient data is compared with male normal limits by mistake. A defect might be anticipated in which of the following regions? 

 a. Anterior   b. Inferior   c. Lateral  d. Septal  

 Explanation: the normal male file anticipates that inferior count density is slightly lower than that of the anterior wall due to diaphragmatic attenuation. However, in a woman, due to breast attenuation anterior count density is the same (counterbalancing diaphragmatic attenuation) or less than that of the inferior wall. Therefore, comparison of a normal female study to the normal male file will result in an artifactual anterior defect. 

 3. Regional wall motion in the distribution of an exercise stress‐induced perfusion abnormality on stress gated 

perfusion tomograms is most often:  

a. Normal b. Hypokinetic c. Akinetic d. Dyskinetic 

 Explanation: Exercise‐induced regional wall motion abnormalities usually resolve quickly (< 3 minutes). Post‐stress gated image acquisition is usually performed at 30 minutes, and stress‐induced wall motion abnormalities will have resolved by that time. The relatively rare exception is post‐stress stunning. However, when post‐stress stunning is detected it is specific marker of underlying severe coronary artery disease.    4. For SPECT imaging with Tc‐99m sestamibi, compared to SPECT acquisition at 30 minutes following 

radiotracer injection at rest, delayed imaging at 90 minutes will demonstrate an interval decrease in tracer concentration in the  

Page 29: Client Login - EXAM 1 Questions and Answers · 2015-09-29 · 18. Which of the following most affects the technetium yield from a 99Mo‐99mTc generator? a. Channeling b. Radiolysis

a. Heart    b. Liver    c. Stomach    d. Gallbladder 

 Explanation: Liver activity will gradually be excreted via the hepatobiliary tract over time. Sestamibi does not substantially wash out from the heart. Water or food will clear activity from the liver. Fat will cause the gallbladder to contract and excrete tracer into the duodenum.   5. A patient undergoing rest/stress Tc99m sestamibi SPECT demonstrates right hemidiaphragmatic elevation 

and intense radiotracer concentration in the liver in the planar projection images.  SPECT images are processed using filtered backprojection. 

The artifact most likely to be present in the processed tomograms is:   a. Flood field non‐uniformity artifact    b. Center of rotation artifact    c. Ramp filter artifact    d. Attenuation artifact  

 Explanation: With intense tracer concentration in the liver, the Ramp filter (inherent to filtered back projection) will artifactually decrease count density in the x‐plane across the field of view.  

6. A patient undergoing rest/stress Tc99m sestamibi SPECT demonstrates right hemidiaphragmatic elevation and intense radiotracer concentration in the liver in the planar projection images.  SPECT images are processed using filtered backprojection. 

 To minimize the possibility of a ramp filter artifacts in this particular patient, which of the following measures will be MOST helpful?  a. Widen the energy window over the Tc99m 140 KeV photopeak  b. Lengthen the SPECT acquisition time by 50% c. Substitute iterative reconstruction for filtered backprojection    d. Ask the patient to drink 16 oz. of water and repeat the SPECT acquisition in 10 minutes 

 Explanation: IR will minimize the Ramp filter artifact. Widening the energy window will INCREASE the amount of scattered photons accepted and thus accentuate the Compton scatter artifact. Water will help to clear radiotracer from the stomach, but will not hasten lever clearance.  7.   A patient undergoes resting myocardial perfusion SPECT using the following protocol: 

(1)  SPECT acquired 40 minutes post 8mCi Tc‐99m sestamibi (2)  High resolution parallel holes (HRP) collimator (3)  Dual‐headed 90˚ angled scintillation detectors, 180˚ acquisition (4)  Filtered back projection; Butterworth filter, cut‐off frequency 0.40, order=10 

 Which of the following will increase image count density?  a. Substitute Tl‐201  b. Substitute a LEAP collimator  c. Decrease the cut‐off frequency to 0.32    d. Decrease the order to 5  

 Explanation: Image processing (changing the cut‐off frequency or order) will not change image count density.  Thallium (3‐4 mCi dose) will yield lower count density than a Tc‐99m tracer. A LEAP collimator has a sensitivity >2 as compare to a HRP collimator, and thus will increase count density.  8.   A patient undergoes resting myocardial perfusion SPECT using the following protocol: 

(1)  SPECT acquired 40 minutes post 8mCi Tc‐99m sestamibi   (2)  High resolution parallel holes (HRP) collimator 

Page 30: Client Login - EXAM 1 Questions and Answers · 2015-09-29 · 18. Which of the following most affects the technetium yield from a 99Mo‐99mTc generator? a. Channeling b. Radiolysis

  (3)  Dual‐headed 90˚ angled scintillation detectors, 180˚ acquisition (4)  Filtered back projection; Butterworth filter, cut‐off frequency 0.40, order=10 

 Which of the following will most likely change the appearance of LV cavity size?  a. Substitute Tc 99m tetrafosmin b. Using attenuation correction c. Use of a single‐headed camera    d. Decrease the cut‐off frequency to 0.25  

 Explanation: A smoother filter will all decrease spatial resolution, making the LV cavity size appear smaller. Assuming the same inherent camera resolution and the same image acquisition time per stop, resolution with a single‐headed vs. dual‐headed camera will be the same. The resolution should not change with tetrofosmin, another Tc‐99m radiotracer. Attenuation correction may slightly improve resolution, but usually has no effect. 

 9.  Inclusion of the descending thoracic aorta in the background ROI of a planar equilibrium radionuclide ventriculography study will result in the following error:  

a. Decrease LV EDV  b. Increase LV EDV and ESV    c. Increase LVEF    d. Decrease LVEF  

 Explanation: Background activity is the subtrahend of the denominator of the LVEF equation. Increasing background will therefore increase LVEF. 

 10.  Which one of the following parameters are routinely quantified with gated blood pool SPECT but not with planar equilibrium radionuclide ventriculography  

a. Left ventricular ejection fraction b. Left ventricular volume c. Left ventricular peak emptying rate d. Left ventricular peak filling rate 

 Explanation: Planar RNV determines LVEF, PER, and PFR by quantifying LV blood pool counts, not actual volumes. Whereas LV volumes are very difficult or impossible to determine from planar equilibrium  RNV, LV volume is routinely determined three‐dimensionally with commercially available gated blood pool SPECT software. Gated blood pool SPECT determines LV volumes by edge detection of the LV blood pool three‐dimensionally. 

 Due to the poorer temporal resolution of gated blood pool SPECT as compared to planar equilibrium radionuclide ventriculography (16 versus 32 frames per cardiac cycle), filling and emptying rates are less accurate.  11.  Evaluation of RVEF with first pass radionuclide ventriculography can be improved by:  

a. Rapidly injecting the radiotracer bolus    b. Slowly injecting the radiotracer bolus    c. Asking the patient to breath‐hold during the radiotracer bolus injection    d. Inject the radiotracer bolus with the patient at end‐expiration    

 Explanation: A rapid, compact bolus may pass so rapidly through the RV that only one, or possibly two cardiac cycles are available for analysis. A slower bolus will provide more cycles for analysis. 

 12.  In a patient scheduled for chemotherapy for breast cancer, a baseline MUGA scan demonstrated an LVEF=69%. For a follow‐up MUGA scan, which of the following meets the criterion for chemotherapy cardiotoxicity?  

a. LVEF =65% b. LVEF=59% 

Page 31: Client Login - EXAM 1 Questions and Answers · 2015-09-29 · 18. Which of the following most affects the technetium yield from a 99Mo‐99mTc generator? a. Channeling b. Radiolysis

c. LVEF=55% d. LVEF=49%  

 Explanation: A decrease in LVEF by 10 points and an absolute value <50% are the criteria for cardiotoxicity in patients with baseline LVEFs > 50%.  Schwartz RG et al. American J. Med. 82: 1109‐1118, 1987  

 13.  Of the following methods, which provides the most accurate assessment of left ventricular peak filling rate?  

a. Planar equilibrium radionuclide ventriculography b. SPECT equilibrium radionuclide ventriculography c. Gated myocardial perfusion SPECT d. CT contrast ventriculography 

 Explanation: Planar equilibrium radionuclide ventriculography has the highest temporal resolution (32 frames per cardiac cycle) compared to 8‐16 frames/cycle for the other methods. Higher temporal resolution allows for a more accurate assessment of dV/dT. 

 14.  The best projection for evaluation of the inferior wall with a planar ERNA scan is the  

a. 45‐degree LAO b. “Best septal” LAO c. Anterior d. Left lateral 

 Explanation: The inferior wall is obscured by the apex in the LAO and best‐septal LAO views and by the RV in the anterior view, but is usually well defined in the left lateral view.  

 LAO (left), left lateral (middle), and anterior (right) equilibrium radionuclide ventriculography views. The inferior wall is best visualized in the left lateral view. 

  

15.  Attenuation correction of Tc‐99m myocardial SPECT is most robust using which of the following emission sources?  

a. Co‐57 flood fields b. Tc‐99m scanning sources c. Gd‐153 rods d. Low‐dose x‐ray sources  

Explanation: Gadolinium‐153 scanning line sources were the first commercially available method to perform cardiac SPECT attenuation correction but yielded low‐flux transmission scans. Current low‐dose x‐ray sources yield higher flux, more accurate transmission scans for attenuation correction.  

 16.  Attenuation correction is most accurate for myocardial perfusion SPECT performed using a myocardial perfusion radiopharmaceutical labeled with  

a.    Tc‐99m b.    Tl‐201 c.    Ga‐67 d.    I‐123 

 Explanation: Vendors have optimized depth‐dependent resolution recovery and scatter correction primarily for Tc‐99m. 

 

Page 32: Client Login - EXAM 1 Questions and Answers · 2015-09-29 · 18. Which of the following most affects the technetium yield from a 99Mo‐99mTc generator? a. Channeling b. Radiolysis

17.  Data dropout in the last bin of an 8‐bin ECG gated myocardial perfusion SPECT scan is due to   

a. Arrhythmic beat rejection b. R‐R interval variability of accepted cardiac cycles c. “Forward‐backward” gating d. Intermittent gating of both the R wave and T wave 

 Explanation: All accepted cardiac cycles are “binned”, with data immediately following the R‐wave assigned to bin #1, and subsequent temporal data assigned to bins #2‐#8. If the R‐R interval is variable, shorter cardiac cycles will not fill all of the bins, particularly bin#8.   18.  Performing a 16‐frame per cardiac cycle gated myocardial perfusion SPECT scan rather than an 8‐frame study would   

a. Increase calculated LVEF  b. Decrease calculated LVEF c. Decrease the number of rejected cycles d. Increase the number of rejected cycles 

 Explanation: Higher number of gated frames results in higher temporal resolution, with larger end diastolic volumes and smaller end systolic volumes and a higher LVEF.  Navare et al Eur J Nucl Med Mol Imaging (2003) 30:1330‐1337 

 19.  Tc‐99m red blood cell tagging efficiency is the poorest using which of the following methods:  

a. Tagging by means of the commercially available UltraTag kit b. In vitro tagging  c. Modified in vivo tagging d. In vivo tagging 

 Explanation: Compared to the in vivo and modified methods, in vitro tagging avoids non‐specific tagging of proteins and other molecules throughout the body with stannous ion, and also prevents excretion of free pertechnetate by the kidneys as well as uptake by the thyroid, and stomach, thereby improving tagging efficiency.  Modified in vivo tagging avoids competition of the kidneys, thyroid, and stomach for injected Tc‐99m pertechnetate.  

 20. Artifact associated with attenuation correction is most likely to result in decreased sensitivity in detecting a perfusion defect in which of the following walls?    

a. Anterior b. Septal c. Lateral d. Inferior 

 Explanation: Attenuation correction results in more apparent scatter from subdiaphragmatic structures, increasing inferior wall myocardial count density and potentially obscuring inferior perfusion defects. 

  

21. Which of the following artifacts is most readily anticipated by careful inspection of the MPI SPECT sinogram? 

 a. Flood field non‐uniformity b. Ramp filter artifact c. Horizontal patient motion d. Vertical patient motion 

 Explanation: The lateral and septal epicardial borders form the lateral and medial borders of the sinogram, respectively. Horizontal motion results in discontinuity of these borders. 

Page 33: Client Login - EXAM 1 Questions and Answers · 2015-09-29 · 18. Which of the following most affects the technetium yield from a 99Mo‐99mTc generator? a. Channeling b. Radiolysis

 Sinogram before (left) and after (right) correction of horizontal patient motion  22.  Compared to standard sodium iodide detector cameras, with the currently available cadmium‐zinc‐telluride 

solid state cameras, it is more difficult to identify which of the following artifacts: 

 

a. Patient motion b. Breast attenuation c. Center of rotation artifact d. Photomultiplier tube malfunction 

 Explanation:   23. If a camera with the flood field below were used, which of the following artifacts would most likely be 

present in reconstructed myocardial perfusion SPECT images? 

  

a. Truncation artifact b. Center‐of‐rotation artifact c. Ring artifact d. Ramp filter artifact e. Partial volume artifact 

 Explanation: As flood field defects are projected over 360° or 180°, they create “hot” or “cold” rings in the reconstructed SPECT images. 

   24.  Factors influencing loss of spatial resolution with depth for myocardial perfusion SPECT are primarily related 

to  

 

a. Hydration of the sodium iodide crystal b. The center of rotation c. Collimator characteristics d. Flood field uniformity  

Page 34: Client Login - EXAM 1 Questions and Answers · 2015-09-29 · 18. Which of the following most affects the technetium yield from a 99Mo‐99mTc generator? a. Channeling b. Radiolysis

Explanation: Loss of resolution with depth is related to the length, width, and degree of septal penetration of the collimator holes. With short, wide holes the loss of resolution with depth is more pronounced that with a collimator with long, narrow holes.  25.  An obese 44yo female underwent a single‐day 10mCi rest/31mCi stress Tc‐99m tetrofosmin protocol. In 

SPECT images the observed stress/rest myocardial count density ratio was 2:1`. The least likely explanation for 

this finding is: 

 

a. The stress radiopharmaceutical dose was partially infiltrated b. The patient developed a severe arrhythmia during the stress SPECT acquisition c. An ECG lead came loose half way through the stress SPECT acquisition d. The position of the patient’s left breast differed considerably in the rest versus the stress SPECT 

acquisition  

Explanation: Decreased image count density can occur if the dose is partially infiltrated or spilled and if beats are rejected due to arrhythmia or loose ECG leads. Irregular beats are rejected as are counts that normally would be acquired during those intervals. Attenuators can decrease count density but would not be expected to decrease counts so far (a 2:1 ratio) below the normal >3:1 stress/rest count density ratio.  26. Compared to standard sodium iodide detector cameras, with the currently available cadmium‐zinc‐telluride 

solid state cameras acceptable quality myocardial perfusion SPECT can be achieved using: 

 

a. Half time/half dose b. Less than half the standard SPECT acquisition time c. Either a or b d. Neither a nor b 

 Explanation: With modern solid state CZT detectors adequate SPECT image quality can be achieved with ¼ the standard counting statistics. This allows for a variety of combinations of reduced injected activity and/or reduced acquisition time to provide patient‐centered imaging.  27.  The sinogram below was obtained in which one of the following patients? 

 

   a. 56 yo woman with arrhythmia and considerable irregular beat rejection b. 65 yo obese woman with large pendulous breasts c. 80 yo man undergoing pharmacologic stress with marked persistence of tracer concentration in the liver d. 66 yo man with partially infiltrated radiopharmaceutical dose 

 Explanation: The sinogram is a “stack” of planar projection images, each collapsed in the y‐axis. These compressed images are stacked one below another from the 45° RAO projection to the 45° LPO projection. In this case there is a marked decrease in myocardial count density in the lower half of the sonogram, representing the LAO‐to‐LPO projections. Large, pendulous breasts are often in this location and can result in marked attenuation and decreased count density.    

Page 35: Client Login - EXAM 1 Questions and Answers · 2015-09-29 · 18. Which of the following most affects the technetium yield from a 99Mo‐99mTc generator? a. Channeling b. Radiolysis

28.  A single‐day Tc‐99m sestamibi rest/stress myocardial perfusion SPECT study was performed using new, 

reduced count density software. Resting activity administered was 6 mCi, and a 12‐minute SPECT acquisition 

was performed. The most appropriate subsequent stress activity and acquisition time are: 

 a. 18 mCi, 10 min b. 12 mCi, 12 min c. 6 mCi, 12 min d. 6 mCi, 18 min 

 Explanation: Irrespective of software or hardware, for a single‐day rest/stress protocol the injected activity ratio should be 3:1 or greater in order to prevent rest‐to‐stress “shine‐through”.  29.  For a two‐day stress/rest Tc‐99m sestamibi protocol both the stress and rest SPECT acquisitions were gated. 

Both the stress and rest perfusion images were entirely normal. The following functional parameters were 

determined: 

  REST  STRESS 

EDV  95 cc  96 cc 

ESV  37 cc  45 cc 

EF  61%  53%  These findings are most consistent with:   

a. Balanced ischemia b. Ischemic cardiomyopathy c. Non‐ischemic cardiomyopathy d. ECG gating error 

 Explanation: An increase in ESV and decrease in LVEF during stress that persists at the time of post‐stress SPECT acquisition suggests post‐stress stunning. With normal myocardial perfusion this is most compatible with stress‐induced severe balanced ischemia. 

 30.  In a gated myocardial perfusion SPECT scan considerable variation in the R‐R intervals of the accepted 

cardiac cycles occurred. From reconstructed gated images which portion of the LV volume curve will most likely 

be affected? 

 

a. Pre‐ejection period b. Rapid systolic emptying c. End systole d. Early/mid diastolic filling e. The atrial kick  

 Explanation:  An average R‐R interval for the patient is determined immediately prior to SPECT image acquisition. Beats within that average R‐R interval + 50% are accepted. With gating, volume curves for each of the acquired cardiac cycles are initiated at the R‐wave. Accepted beats that are short therefore do not fill the entire prescribed R‐R interval. Therefore, when all the volume curves for the acquired cardiac cycles are summed, there is data “dropout at the end/diastolic portion of the LV volume curve. 

 31.  Post‐stress and resting Tc‐99m sestamibi myocardial perfusion SEPCT short axis tomograms are shown 

below. The TID ratio is 0.32. 

 Top row = stress Bottom row = rest 

Page 36: Client Login - EXAM 1 Questions and Answers · 2015-09-29 · 18. Which of the following most affects the technetium yield from a 99Mo‐99mTc generator? a. Channeling b. Radiolysis

   Findings are most consistent with:  

a. Balanced ischemia b. Resting ischemia c. Tachycardia during stress acquisition d. Left bundle branch block 

 Explanation: In this case the LV, particularly the LV cavity, is smaller in the post‐stress SPECT images than in the resting images, thus “reverse TID”. LV volume is influenced by heart rate. At higher heart rates there is less time for diastolic filling and therefore the heart is smaller.   32.  Which of the following parameters are more easily quantified with gated blood pool SPECT but not with equilibrium radionuclide ventriculography 

a. Left ventricular ejection fraction b. Left ventricular volume c. Right to left shunt evaluation d. Right ventricular volume  Explanation: From an anterior or RAO planar image the LV volume is determined planimetrically from first   pass RNV. LVEF and RVEF can be determined by time‐activity curves from regions of interest over the LV and RV respectively. Whereas RV volumes are very difficult or impossible to determine by first pass RNV, RV volume is routinely determined three‐dimensionally with commercially available gated blood pool SPECT software.   

33. When the phase analysis image is displayed, pixels with the same color represent: 

 

a. Same amplitude 

b. Same stroke volume 

c. Temporal synchrony 

d. Wall motion abnormalities 

Explanation: The typical display of the phase image, codes pixels whose contractility occurs at the same time in the cardiac cycle, with like color.     34.  Using nuclear techniques, a peak stress ejection fraction can be acquired by:     

a. Planar ERNA and first‐pass RNA  b. Planar  ERNA and gated SPECT MPI    c. SPECT ERNA and first‐pass RNA    d. SPECT ERNA and SPECT MPI   

Explanation: SPECT MPI images are acquired post‐stress. SPECT images are acquired over a 12‐20 minute period.  Stress MUGA acquisitions are 2‐3 minutes, while those for first‐pass RNAs are 20‐30 seconds allowing for capture of peak stress images using these techniques.  

Page 37: Client Login - EXAM 1 Questions and Answers · 2015-09-29 · 18. Which of the following most affects the technetium yield from a 99Mo‐99mTc generator? a. Channeling b. Radiolysis

 35.  The LV volume curve gives information about all of the following, except:  

a. Beat rejection b. Gating accuracy c. Systolic function d. Diastolic function 

   36.  When performing a first‐pass radionuclide angiogram to evaluate the left ventricle, the radionuclide should be drawn up in a volume of:  

a. 0 ‐ 1 ml    b. 2 – 4 ml    c. 5 ‐ 10 ml    d. 11 ‐ 20 ml   

Explanation: Since images are acquired as a bolus of activity passes through the heart and lungs, it is critical to administer the activity in a small volume.  Most sources suggest less than 0.5 ml and the others, less than 1ml.  37.  Which of the following is a potential advantage of SPECT‐ERNA over planar ERNA: 

 

a. The detector is further from the patient’s chest 

b. SPECT filters improve temporal resolution 

c. Better visualization of LV and RV separation 

d. Acquisition is less dependent on regular R‐R intervals 

Explanation: In tomographic ERNA, there are more views IE SAX, VLA and HLA, giving a better view of the septum. A better view of the septum yields better estimation of L and R ventricular separation.  38.  When using an in‐vivo RBC tagging method, Tc‐99m sodium pertechnetate should be injected how long after stannous pyrophosphate? 

a. 5 minutes b. 15 minutes     c. 45 minutes d. 60 minutes 

 Explanation: According to ASNC guidelines, when using an in‐vivo RBC tagging method, Tc‐99m sodiumpertechnetate should be injected 15 minutes after stannous pyrophosphate.     

     

Page 38: Client Login - EXAM 1 Questions and Answers · 2015-09-29 · 18. Which of the following most affects the technetium yield from a 99Mo‐99mTc generator? a. Channeling b. Radiolysis

 

American Society of Nuclear Cardiology 2015 Nuclear Cardiology Board Exam Preparation Course 

Washington, DC  

EXAM 4 – Questions and Answers   

 Topics Covered: The Basics of Radiation Safety Instrumentation Quality Control Assessment of Myocardial Viability Including PET  1. A restricted area where radiation levels could result in an individual receiving a dose equivalent of >5mRem 

in an hour at 30cm from the radiation source would warrant the following warning sign.  

a. Caution, Radioactive Materials b. Caution,  Airborne Radioactivity Area c. Caution, Radiation Area d. Caution, HIGH Radiation Area  

Explanation: By definition, A “radiation area” sign is needed when individuals could receive in excess of 0.005Rem in 1 hour at 30cm from a source or any surface where radiation penetrates  (High Radiation Area ‐ >100mRem in 1 hour @ 30cm from a source).  2. Daily uniformity tests should acquire at least ____________ counts.  

a. 500,000 b. 1,000,000 c. 3,000,000   d. 10,000,000 

 3. Patient's administered with radioactive material may be discharged from the nuclear department providing 

they are not likely to expose any individual to an excess of _____mRem.  

  a.  50   b.  100   c.  500   d.  5000  NUREG 1556, Volume 9 Appendix U  Explanation: 10 CRF Part 35 permits a licensee to release from its control an individual who has been administered with unsealed  byproduct material if the total effective dose equivalent to any other individual, from the exposure of the released individual, is not likely to exceed 5mSv or 0.5 Rem.   4. Which of the following quality control procedures should be performed quarterly? 

a. Energy peaking b. Uniformity testing c. SPECT phantom d. Resolution and four quad linearity phantom  

Explanation:   Energy peaking and uniformity  testing are daily procedures.   Planar  resolution and  linearity  (bar) phantoms are weekly procedures.  Plexiglass (Jaszczak) phantoms are a quarterly procedure. 

   

Page 39: Client Login - EXAM 1 Questions and Answers · 2015-09-29 · 18. Which of the following most affects the technetium yield from a 99Mo‐99mTc generator? a. Channeling b. Radiolysis

5. The most significant effect of a longer positron range on image quality is on:  

a. Spatial resolution 

b. Attenuation correction 

c. Count density 

d. Acquisition time 

Explanation: Positron range is the distance travelled by the positron before it annihilates with an electron to release the two gamma rays. Radiotracers with longer positron range have worse spatial resolution compared to radiotracers with a shorter positron range. Positron range does not affect attenuation correction, count density, or acquisition time.  References: Di Carli MF et al. J Nucl Med. 2007;48:783‐793. 

 6. Which of the following isotopes warrants prolonged (>48 hours) interruption of breast feeding following 

nuclear imaging?  

a. I‐123 b. Rb‐82 c. TI‐201 d. Tc‐99m  

Explanation: For 99m‐Tc, it is recommended  to stop breastfeeding for 48 hours, whereas for 201‐Tl it can be up to 7 weeks, thus making  prolonged breastfeeding interruption a necessity  if a Thallium study is performed . !‐131 not 1‐123 requires also prolonged interruption of breast feeding.  

 7. Conventional SPECT imaging in the prone compared to the supine position results in:  

a) Greater reconstruction artifact b) Greater lateral wall attenuation c) Less inferior wall attenuation d) Greater motion artifact 

 Explanation: Prone imaging has been reported to produce less patient motion and less inferior wall attenuation than supine imaging. Prone imaging results in higher specificity for RCA disease than supine imaging. The combination of supine and prone images may also be helpful in identifying attenuation artifacts due to breast and/or excessive lateral chest‐wall fat, due to the shift in position of the attenuating structures that occur in the prone position. Prone imaging does not eliminate attenuation artifact but rather simply changes the location. By comparing supine and prone images, artifactual defects will change their location whereas true perfusion defects will remain fixed.  References: 

Segall GM, Davis MJ. Prone versus supine thallium myocardial SPECT: A method to decrease artifactual 

inferior wall defects. J Nucl Med 1989;30:548‐55. 

Kiat H, Van Train KF, Friedman JD, Germano G, Silagan G, Wang FP, et al. Quantitative stress‐redistribution 

thallium‐201 SPECT using prone imaging: Methodologic development and validation. J Nucl Med 

1992;33:1509‐15. 

Nishina H, Slomka PJ, Abidov A, et al. Combined supine and prone quantitative myocardial perfusion SPECT: 

Method development and clinical validation in patients with no known coronary artery disease. J Nucl Med 

2006;47:51‐8. 

Slomka PJ, Nishina H, Abidov A, et al. Combined quantitative supine‐prone myocardial perfusion SPECT 

improves detection of coronary artery disease and normalcy rates in women. J Nucl Cardiol 2007;14:44‐52. 

 

Page 40: Client Login - EXAM 1 Questions and Answers · 2015-09-29 · 18. Which of the following most affects the technetium yield from a 99Mo‐99mTc generator? a. Channeling b. Radiolysis

8. Sievert is a unit of :  

a. Exposure b. Equivalent dose c. Dose rate d. Activity  Explanation: The SI unit for exposure is C/kg.  The SI unit for activity is Bequerel (Bq).  

9. Exercise myocardial perfusion imaging with PET is best performed using:  

a. N‐13 ammonia  b. Rubidium‐82  c. F‐18 FDG  d. O‐15 water   

Explanation: PET radiotracers have much shorter half‐lives than SPECT radiotracers, and are best used with pharmacological stress. The half life of PET tracers are: Rubidium‐82 (75 seconds), O‐15 water (2 minutes), N‐13 ammonia (10 minutes) and F‐18 FDG (110 minutes). Rubidium‐82 and O‐15 water are best suited for pharmacological stress; N‐13 ammonia can be used with exercise or pharmacological stress; although F‐18 FDG can be used with exercise stress, it images ischemic memory, and not myocardial perfusion. 

 Reference: Dilsizian V, Bacharach,S.L., Beanlands, R.S., Bergmann, S.R., Delbeke, D., Gropler,R.J., Knuuti, J., Schelbert,H.R., Travin, M. PET myocardial perfusion and metabolism clinical imaging. In: http://wwwasncorg/imageuploads/ImagingGuidelinesPETJuly2009pdf; 2008. 

 10. Bar phantoms are used to evaluate resolution in which of the following?  

a. planar uniformity b. planar linearity    c. SPECT uniformity d. SPECT linearity  

Explanation: Bar phantoms are used to evaluate planar linearity and resolution.  Daily floods evaluate planar uniformity.  SPECT contrast, resolution and uniformity are evaluated with refillable Plexiglas phantoms.  11. If the estimated effective dose for a rest and stress cardiac SPECT study is 12 mSv, this can be explained to 

the subjects as how many times the yearly back ground exposure?  

a. 2 times  b. 4  times     c. 10  times   d. 20  times     

Explanation: The correct answer is approximately 4 times the natural background exposure (background exposure = 3 mSv).  12. A “tuning fork” artifact is caused by a bad:  

a. uniformity flood b. bar phantom c. center of rotation   d. Jaszczak phantom 

 Explanation:  Tuning fork (180) and donout (360) artifact are indicative of a center of rotation misalignment. 

  

13. Area surveys must be performed: 

Page 41: Client Login - EXAM 1 Questions and Answers · 2015-09-29 · 18. Which of the following most affects the technetium yield from a 99Mo‐99mTc generator? a. Channeling b. Radiolysis

 a. at the beginning of each working day   b. at the end of each working day  c. weekly  d. monthly  

Explanation:  Daily surveys should be completed the END of each day of use, where unsealed radioactive material is prepared for use or administered.  14. Radioactive packages must be monitored within what period following delivery?  

a. Immediately  b. 1 working hour  c. 2 working hours    d. 3 working hours     

Explanation:  The package should be surveyed and wipe tested within 3 hours of receipt if during normal working hours. If the package is delivered while the facility is closed, it should be surveyed and wipe tested within 3 hours of the next business day. 

 15. Uniformity tests should be performed:  

a. daily     b. weekly c. monthly d. quarterly 

 Explanation: Uniformity test should be performed daily! 

 16. The maximum occupational exposure per year to a nuclear medicine technologist is:  

a. 5 rem/year whole body    b. 5 rem/lens of eye   c. 5 rem/year to testes   d. 5 rem/year to the hand    

Explanation: The NRC (Nuclear regulatory Commission) recommends this threshold for occupational workers based on the principles of ALARA, (as low as reasonably achievable) (http://www.nrc.gov/reading‐rm/doc‐collections/cfr/part020/part020‐1201.html.)   

 17. Patient's administered with radioactive material may be discharged from the nuclear department providing 

they are not likely to expose any individual to an excess of _____mRem.   

a.  50 b.  100 

  c.  500   d.  5000  NUREG 1556, Volume 9 Appendix U  Explanation: 10 CRF Part 35 permits a licensee to release from its control an individual who has been administered with unsealed  byproduct material if the total effective dose equivalent to any other individual, from the exposure of the released individual, is not likely to exceed 5mSv or 0.5 Rem.   18.  (a) The rubidium‐82  rest stress PET acquisition (shown below)  was obtained from a patient with an 

ejection fraction of 10% using the acquisition protocol following the ASNC Imaging Guidelines for Nuclear 

Cardiology procedures for a patient with a normal ejection fraction. What is the most likely explanation for 

the poor image quality?  

Page 42: Client Login - EXAM 1 Questions and Answers · 2015-09-29 · 18. Which of the following most affects the technetium yield from a 99Mo‐99mTc generator? a. Channeling b. Radiolysis

 a.  Patient motion b.  Body habitus c.  Excessive blood pool counts d.  An artifact e.  Low dose of isotope 

  

18 (b). The  patient described above is brought back for a repeat  rubidium‐82 study the next day. Which of the following parameters should be modified to improve image quality of the above scan after radiotracer injection? 

 a.  increase  rest‐stress radiotracer dose ratio b.  use 3D PET imaging c.  rescan the patient in 30 minutes d.  prolonged initial scan delay  

 

Learning Objective: To tailor imaging parameters of PET to account for lower ejection fraction and delayed distribution uptake of Rb‐82. Thus by prolonging intial scan delay allows blood pool clearance of rubidium‐82 in patients with poor LV systolic function ( given prolonged circulation time )  Discussion:  In patients with normal LV function, imaging should begin 70‐90 seconds after Rb‐82 infusion.  When imaging patients with very poor ejection fractions (i.e., less than 30%), this time should be increased to 110 to 130 seconds after injection of Rb‐82. Very poor ejection fractions can delay distribution and uptake of Rb‐82, causing excessive blood pool counts.  Reference: Dilsizian V, Bacharach SL, Beanlands RS, et al. Imaging guidelines for nuclear cardiology procedures: PET myocardial perfusion and metabolism clinical imaging. J Nucl Cardiol. 2009; 16; doi:10.1007/s12350‐009‐9094‐9. http://www.asnc.org/imageuploads/ImagingGuidelinesPETJuly2009.pdf   

Page 43: Client Login - EXAM 1 Questions and Answers · 2015-09-29 · 18. Which of the following most affects the technetium yield from a 99Mo‐99mTc generator? a. Channeling b. Radiolysis

 19. Extrinsic uniformity of a gamma camera is evaluated: 

a. With a collimator b. Without a collimator c. Using a point source d. Using a bar phantom 

 Explanation:    Extrinsic  is with  the  collimator.    Extrinsic  procedures  are  performed  using  a  sheet  source.    Point sources are used when performing intrinsic evaluations (without a collimator).  Bar phantoms are used to evaluate linearity and resolution, not uniformity.  20. A spill of at least how many mCi of Tc99m is defined as a major spill? 

  a.  10   b.  30   c.  50   d.  100  NUREG 1556, Volume 9 Appendix N  21. A survey meter should be calibrated 

a. Daily b. Weekly c. Monthly d. Annually 

 

Explanation: It is a requirement that the survey meter is calibrated annually to ensure that the meter responds 

accurately to radiation.  The daily QC is a source check only verifying that the instrument responds to a radioactive 

source. 

 

22. Which of the following is a standard procedure for radioactive waste disposal by decay‐in‐storage? 

   a. Removal of all radioactive material labels   b.  Ensure shielding of the container    c.  Maintain waste disposal records for 5 years   d.  Ensure the surface of the container does not exceed 0.5mR/hr on any one side   NUREG 1556, Volume 9 Appendix W  Prior to disposal as in‐house waste, all radioactive material labels must be removed or defaced.  Explanation of other answers: b.  While in storage waste should be stored in properly shielded container c.  Records of disposal must be maintained for 3 years (10 CRR 35.2092) d.  Prior to disposal, waste must be indistinguishable from background    

 23. A daily uniformity flood is used to detect which of the following: 

 a. A malfunctioning photomultiplier tube b. Center of rotation error c. Poor resolution d. Increased dead time of the NaI (Tl) crystal 

 Explanation:   Daily uniformity  testing  looks  for any non‐uniformity across  the  field of view.   To evaluate planar resolution a bar phantom is used.  SPECT resolution would be evaluated by using a plexiglass (Jaszczak) phantom.  Center of rotation testing is done by doing SPECT imaging of a radioactive point source.   

Page 44: Client Login - EXAM 1 Questions and Answers · 2015-09-29 · 18. Which of the following most affects the technetium yield from a 99Mo‐99mTc generator? a. Channeling b. Radiolysis

 24. F‐18 FDG enters the myocyte via the following mechanism: 

 

a. Active uptake via Na/K ATPase pump 

b. Passive diffusion 

c. Active extraction via K+ channels 

d. Glucose transporters 

Explanation:  Option D, glucose transporters is correct. F‐18 FDG enters the myocyte through facilitated diffusion via the glucose transporters, most commonly, Glut 4 transporter. Insulin acutely stimulates glucose transport in muscle by recruiting up to about 40% of cytosolic GLUT 4 to the cell membrane (normally < 1%), increasing glucose uptake by about 10–40 fold. GLUT 1 translocation may also be triggered by insulin. Options A, B, and C are incorrect.  References: King LM and Opie LH. Molecular and Cellular Biochemistry 180: 3–26, 1998. 

 25.  A 45 year old male with history of uncontrolled hypertension presents with progressive shortness of breath with exertion of 2 months duration.  His presenting blood pressure was 160 /90 and he has not taken any medications for over 6 months due to financial issues. His electrocardiogram shows normal sinus rhythm with nonspecific ST T waves changes. His troponins are negative over a period of 8 hours. He undergoes pharmacological rest stress rubidium‐82 perfusion PET. Resting EF on PET was 38% and peak stress EF was 36% with generalized mild global hypokinesis.  PET images are shown below   

 

Which of the following is the most appropriate next step? 

a) Cardiac catheterization 

b) Dobutamine echocardiogram 

c) Medical management 

d) MUGA scan  

Learning Objective: recognize SPECT patterns of non‐ischemic cardiomyopathy  and appropriate management  

Page 45: Client Login - EXAM 1 Questions and Answers · 2015-09-29 · 18. Which of the following most affects the technetium yield from a 99Mo‐99mTc generator? a. Channeling b. Radiolysis

Discussion: The SPECT scan shows a dilated ventricle with predominantly normal myocardial perfusion except for 

mild inferior wall reduction in perfusion minimally reversible  which likely could be from variable  attenuation /GI 

scatter. There are no large fixed or reversible perfusion defects to indicate ischemic etiology of cardiomyopathy. 

Gated study show no regional wall motion abnormalities and only diffuse generalized hypokinesis. This along with 

the young age of patient, exercise capacity and negative EKG and uncontrolled hypertension favors dilated non‐

ischemic cardiomyopathy pattern. Focus should be on aggressive control of risk factors mainly hypertension as 

improvement of LV function can occur in such settings. No further workup is indicated at present  

Ref : Danias PG Am J Cardiol. 2004 Jul 1;94(1):14‐9.  

26. Which of the following is NOT a test performed on a dose calibrator?  

a. Accuracy 

b. Precision   

c. Constancy 

d. Geometry 

 

Explanation: Dose calibrator tests include accuracy, constancy, geometry and linearity. 

 27. A radioactive source produces 90 mrem/ hour at a distance of 1 meter from the source. What is the measured exposure at a distance of 3 meters?  

a. 10 mrem/hour  b. 15 mrem/hour   c. 30 mrem/hour    d. 45 mrem/hour    

 28. A 70‐year‐old  female with a history of diabetes presents  for  evaluation of progressive dyspnea on exertion of 2 weeks duration. Two years ago she had large anterior myocardial infarction. Coronary angiography showed an occluded LAD,  a 60% RCA and 70 % left circumflex artery lesions .   After a failed attempt at primary angioplasty of the left anterior descending artery,  she has been treated with aspirin, betablockers and ACE inhibitors.  Her presenting  electrocardiogram in the office  demonstrates sinus rhythm with q waves V1‐V4.  An echocardiogram performed shows  moderate‐severe global left ventricular (LV) dysfunction with a reduced ejection fraction of 30‐35%.  Her creatinine is 2.5 mg /dl. What would be the next most appropriate step?   

a) Cardiac catherization  

b) Rubidium‐82/F18 FDG PET  

c) Intensify medical therapy 

d) Resynchronization therapy 

 Learning objective : Indications of viability study for risk stratification in patients with heart failure and LV dysfunction   Discussion: Despite her age and other medical problems, this is an active patient. Her long‐term prognosis and treatment plan would be best served by knowing whether or not there is enough viable myocardium to justify a revascularization procedure. If no significant viable tissue is present, a cardiac catherization is not warranted in this elderly person. If significant viable tissue is present, this patient may do better with a more aggressive approach..  Ref : HFSA 2010 Heart failure guidelines J Card Fail. 2010 Jun;16(6):e1‐194. doi: 10.1016/j.cardfail.2010.04.004. 

 29. A geometry test on a dose calibrator is performed to:   

a. Assess ability to read both high and low activity     b. Assess ability to read both high and low volumes c. Assess accuracy of readings compared to a known standard 

Page 46: Client Login - EXAM 1 Questions and Answers · 2015-09-29 · 18. Which of the following most affects the technetium yield from a 99Mo‐99mTc generator? a. Channeling b. Radiolysis

d. All of the above  

Explanation: A geometry test is performed to assess ability to read both high and low volumes A linearity test is performed to assess ability to read both high and low activity..  An accuracy test is performed to compare actual readings to decay corrected calibrated activity. 

 30. What is the shallow (extremity) dose annual occupational limit?   

a. 0.5Sv b. 0.15Sv c. 0.05Sv d. 1.5Sv 

 31. Myocardial viability assessment with FDG PET is typically performed with glucose loading and intravenous insulin because this preparation (glucose + insulin) improves glucose uptake in the:  

a. Hibernating myocardium b. Scarred myocardium  c. Stunned  myocardium d. Normal myocardium  

 Explanation: Ischemic myocardium preferentially uses glucose in the fasting as well as the glucose loaded state. Hence, the diagnosis of viable myocardium is not different between fasting or glucose loaded conditions. However, since the normal myocardium switches to glucose metabolism in the glucose loaded state; it serves as a reference segment for comparison of the amount of glucose uptake in the ischemic tissues and improves the quantification of hibernating myocardium.  Also, we are better able to distinguish normal myocardium from scar tissue in the postprandial state (in the fasting state both the normal and scar myocardium will not utilize glucose). Finally, glucose loading proves the homogeneity of the image. 

 Reference: Dilsizian V, Bacharach,S.L., Beanlands, R.S., Bergmann, S.R., Delbeke, D., Gropler,R.J., Knuuti, J., Schelbert,H.R., Travin, M. PET myocardial perfusion and metabolism clinical imaging. In: http://wwwasncorg/imageuploads/ImagingGuidelinesPETJuly2009pdf; 2008.  32. You are returning a Sr/Rb82 Generator to the manufacturer and the surface dose rate is 4.2 mR/hr and is 0.8 mR/hr at one meter.  What radioactive label must be on the box?  

a. White‐I b. Yellow‐II c. Yellow‐III d. No label  

Explanation: Yellow II label is required for packages with a surface dose rate of between 0.5 and 50 mrem/hr, and less than 1.0 millrem/hr at one meter. 

    

Page 47: Client Login - EXAM 1 Questions and Answers · 2015-09-29 · 18. Which of the following most affects the technetium yield from a 99Mo‐99mTc generator? a. Channeling b. Radiolysis

33. A Jaszczak phantom is a:    

a. sheet of embedded lead bars for camera resolution testing b. Co‐57 sheet source for uniformity testing c. plexiglass cylinder for SPECT testing d. hollow phantom for dose calibrator testing  

Explanation: A Jaszczak phantom is a Plexiglas phantom used to evaluate SPECT contrast, resolution and uniformity.  A lead bar phantom is used to evaluate planar resolution and linearity. 

 34. To accurately measure radiation exposure, the most appropriate instrument is the _____________.  

a. Geiger Mueller (GM meter) b. Dose Calibrator c. Ionization Chamber d. Scintillation well chamber 

 Explanation: Dose calibrator and well chamber  are not used to measure exposure. The GM meter is only accurate for the energy it is calibrated (usually Cs137). 

 35. What quality  control  test determines  the  ability of a dose  calibrator  to  compare 10 mCi  Tc99m  in  a 1cc volume compared to 10 mCi Tc99m in a 5cc volume? 

 a. Geometry  b. Linearity c. Accuracy d. Constancy 

 Explanation: GEOMETRY TEST: This test is designed to show that correct readings can be obtained regardless of the volume or geometry. One ml of Tc‐99m in a 10 ml syringe (activity 25 mCi) is measured in the dose calibrator and the value obtained is recorded. The activity is then diluted with water to measure larger volumes. At each of these points  a  reading  is  taken  and  the  value  recorded. Data  are  then  evaluated  to  determine  the  effect  of  sample geometry on the dose calibrator reading.  

 36. The half value layer of lead for a positron emitter is 0.7 cm.  If the dose rate measured outside of an unshielded storage cabinet is 100 mR/hr, how much lead must be added to reduce it to less than 5 mR/hr?  

a. 1.4 cm b. 2.0 cm c. 2.4 cm d. 3.2 cm  

Explanation:  Slightly more than 4 half value layers (> 2.8 cm) are required to reduce the dose rate from 100 to 5 mR/hr. All other thicknesses are not sufficient. 

 37. A linearity test on a dose calibrator is performed to: 

 

a. Assess ability to read both high and low activity     

b. Assess ability to read both high and low volumes 

c. Assess accuracy of readings compared to a known standard 

d. All of the above 

 Explanation: A linearity test is performed to assess ability to read both high and low activity.  A geometry test is performed to assess ability to read both high and low volumes.  An accuracy test is performed to compare actual readings to decay corrected calibrated activity. 

   

Page 48: Client Login - EXAM 1 Questions and Answers · 2015-09-29 · 18. Which of the following most affects the technetium yield from a 99Mo‐99mTc generator? a. Channeling b. Radiolysis

38. The gestational dose limit to a pregnant occupational worker is:  

a. 5 mSv b. 50 mSv c. 0.5Sv d. 0.15Sv  

Explanation: The pregnant employee occupational dose limit is reduced to the non‐occupation dose limit for the gestation period. 

 39. You are exercising someone on the treadmill and spill 15% of the dose of Tc‐99m on the treadmill. You should:  

a. Notify the RSO    b. Report a medical event to the NRC    c. Inform the patient’s physician of a misadministration    d. Repeat the test with a 4 mCi dose of Tl‐201   

Explanation: The usual stress Tc‐99m dose is 30 mCi and 15% spill would equal about 4‐5 mCi. The radiation exposure from that dose is below the occupational and public limits (patient receiving radiation for examination). This degree of spill should be reported to the radiation safety officer but does not mandate reporting to the NRC. This is not a misadministration. The test cannot be repeated with 4 mCi of Tl‐201, because presumably this patient already received Technetium 99‐m, a higher energy radiotracer. 

 40. Deterministic effects of ionizing radiation are those effects that:  

a.  probability increases with dose  b.  are associated with a threshold dose c.  are considered stochastic in nature d.  are responsible for latent signs/symptoms of exposure  

 By definition, deterministic effects have a threshold dose before expression 

 41. An absorbed dose of 10mRad from exposure to a gamma emitter would results in _________mRem.  

a.  200 b.  20 c.  10 d.  1   

Rem = Rad * QF (QF for gamma radiation is 1.0)  

42. Which of the following is the first abnormality in the evolution of normal myocardium to hibernating myocardium? 

 

a. Perfusion 

b. Metabolism 

c. Function 

Explanation:  The correct sequence of abnormalities in the evolution from normal myocardium to hibernating myocardium is reduced perfusion followed by metabolic adaptation and changes in function.  

References: Taegtmeyer H. J Nucl Med. 2010 May 1;51 Suppl 1:80S‐87S. 

43. What percent of photons are transmitted through 2 HVL's (Half Value Layers)?  

a.  25% b.  50% 

Page 49: Client Login - EXAM 1 Questions and Answers · 2015-09-29 · 18. Which of the following most affects the technetium yield from a 99Mo‐99mTc generator? a. Channeling b. Radiolysis

c.  75% d.  100%   

   2 HVL's = 1/4 or 25%  44. If your dose calibrator is not functioning, what method is allowed by regulation to assay a diagnostic radiopharmaceutical prior to injection?  

a. There is no alternative; you must use another dose calibrator b. You can use the activity and assay time provided by the radiopharmacy and apply a decay correction c. You may use your survey instrument provided it was calibrated for the radioisotope you are planning to 

inject d. You may place the dose under a properly operating gamma camera and use an ROI to calculate the 

activity  Explanation: Regulations allow the use of the data provided by a Radiopharmacy to decay correct prior to the administration of a gamma emitting diagnostic radiopharmaceutical. 

  45. A radioactive package surveyed prior to shipping yields an exposure rate of 150mR/hr on the surface.  What DOT label must be used for this package?  

a.  WHITE I b.  YELLOW II c.  YELLOW III d.   Exceeds limits and may not be shipped without special permit   

Yellow III if for packages with a surface exposure of 50mR/hr to 200mR/hr 

  

46. Compared to contractile reserve assessment with low dose dobutamine echo, FDG PET imaging for myocardial viability assessment is 

 a. More sensitive  b. More specific  c. Equally sensitive d. Equally specific  

Explanation: In pooled studies, it is shown that radionuclide techniques are more sensitive and inotropic contractile reserve techniques are more specific for the detection of myocardial viability. 

 References:Camici et al. Stunning, Hibernation, and Assessment of Myocardial Viability. Circulation. 2008;117:103‐114 

   

Page 50: Client Login - EXAM 1 Questions and Answers · 2015-09-29 · 18. Which of the following most affects the technetium yield from a 99Mo‐99mTc generator? a. Channeling b. Radiolysis

47. The following image is  

  a. Uniformity flood b. Linearity and resolution phantom c. Bar phantom d. Jaszczak phantom 

 Explanation: The image is a refillable plexiglass SPECT phantom most commonly known as a Jaszczak phantom. 

 48. Breastfeeding should be interrupted to ensure the infant does not receive greater than ____mRem of radiation from patient.    a.  50   b.  500   c.  5000   d.  background levels  10 CFR 37.75(b)] 

 49. Which of the following dietary preparations is best suited for imaging assessment of sarcoidosis with FDG PET?  

a. high carbohydrate diet b. glucose loading on day of study c. prolonged fasting d. low fat diet e. No dietary restriction 

 Learning Objective: to understand the right patient preparation for a good quality study for diagnosis of cardiac sarcoidosis Discussion : Of the choices provided a prolonged fast is best suited for sarcoidosis assessment with cardiac PET. Fasting shifts myocardial fuel utilization to fatty acids.  In active cardiac sarcoidosis there is predilection for macrophages in the granulomatous lesions to avidly utilize glucose (resulting in uptake of FDG preferentially in these areas) while there is suppression of glucose uptake in rest of myocardium. By comparing patterns of perfusion (Rb‐82 images ) to FDG utilization patterns presence of cardiac sarcoidosis can be detected with high sensitivity specificity 

Ref : Mc Ardle BA et al J Nucl Cardiol. 2013 Apr;20(2):297‐306 

 

 50. The main advantage of using a Geiger‐Mueller detector as a survey meter is it  

Page 51: Client Login - EXAM 1 Questions and Answers · 2015-09-29 · 18. Which of the following most affects the technetium yield from a 99Mo‐99mTc generator? a. Channeling b. Radiolysis

a. can be used to distinguish different types of radiation b. is more sensitive than other gas‐filled detectors c. has less dead time than other gas‐filled detectors d. is typically used as a dose calibrator 

 Explanation: Geiger‐Mueller counters cannot be used to distinguish different types of radiation, because each interaction of radiation with the gas causes maximum ionization; because of this principle, they are much more sensitive than other gas‐filled detectors, but also have much higher dead‐times, requiring the use of a quenching gas or other mechanisms to stop the ionization process; they are typically used as radiation monitors and survey meters. 

 51. If a 20% energy window is set on a centerline of 159 keV (I‐123), the range of photon energies accepted into the image will be  

a. 127 ‐ 201 keV b. 139 ‐ 179 keV c. 143 ‐ 175 keV d. 149 ‐ 169 keV 

 Explanation: A 20% energy window is defined as an energy range of 20% of the centerline, usually centered on the photopeak. 159 keV ± 10% gives 143 ‐ 175 keV. Likewise, at 20% window for the Tc‐99m photopeak of 140 keV is 140 keV ± 14 keV or 126 – 154 keV. ASNC guidelines allow for a 30% window for the 70 keV peak or 70 keV (60 – 80 keV).   Ref. Powsner RA, Plamer MR, Powsner ER. Essentials of nuclear medicine physics and instrumentation, Third Edition. Wiley‐Blackwell. 2013. P. 61. 

 52. In a glucose loaded state, dysfunctionaly myocardium is hibernating if 

 a. Perfusion is reduced and glucose utilization is increased b. Perfusion is normal and glucose utilization is normal c. Perfusion is reduced and glucose utilization is reduced d. Perfusion is normal and glucose utilization is reduced 

 Explanation: A is the correct answer as hibernating myocardium is hypoperfusion myocardium with increased glucose utilization.  B is not correct as it represents non‐ischemic cardiomyopathy C is not correct as it represents scar D is incorrect as it represents stunned myocardium   Reference: Dilsizian V, Bacharach,S.L., Beanlands, R.S., Bergmann, S.R., Delbeke, D., Gropler,R.J., Knuuti, J., Schelbert,H.R., Travin, M. PET myocardial perfusion and metabolism clinical imaging. In: http://wwwasncorg/imageuploads/ImagingGuidelinesPETJuly2009pdf; 2008.  53. Myocardial perfusion defects from mis‐registration between PET emission and transmission images are most commonly dealt with by A. The use of software to correct for the misregistration B. The interpretation of emission images without attenuation correction C. Repeating the transmission scan D. Repeating the emission scan  Explanation: A. is correct. Misregistration of the transmission and emission images can result in artifactual perfusion defects. The best way to deal with mis‐registration between the transmission and emission images is to use software. This software will correctly align the transmission and emission images and develop a new attenuation correction map that is used to generate a new emission image. ASNC recommends that PET images be interpreted with attenuation correction.  

Page 52: Client Login - EXAM 1 Questions and Answers · 2015-09-29 · 18. Which of the following most affects the technetium yield from a 99Mo‐99mTc generator? a. Channeling b. Radiolysis

B is incorrect as non attenuation corrected scans may increase attenuation artifacts in the inferoseptal wall and create a hotspot in the anterolateral walls C and D are incorrect as repeating the transmission and emission scans is not optimal as there is no guarantee that misregistration will be resolved.  Reference: Dilsizian V, Bacharach,S.L., Beanlands, R.S., Bergmann, S.R., Delbeke, D., Gropler,R.J., Knuuti, J., Schelbert,H.R., Travin, M. PET myocardial perfusion and metabolism clinical imaging. In: http://wwwasncorg/imageuploads/ImagingGuidelinesPETJuly2009pdf; 2008.  54. The half value thickness (layer) for 99mTc photons, in tissue is  

a. less than that of 201Tl b. about the same as water c. greater than Pb d. decreases with decrease in Z number 

 Ref: Iskandrian, Ami E. Garcia, Ernest (2008)Eds. Chapt 2, Murphy, Paul H., Radiation Physics and Radiation Safety, p. 22 ‐ 23. 

 

Page 53: Client Login - EXAM 1 Questions and Answers · 2015-09-29 · 18. Which of the following most affects the technetium yield from a 99Mo‐99mTc generator? a. Channeling b. Radiolysis

Jason Tavel The Basics of Radiation Safety Q&A

Thursday, September 18

1. The unit for radiation exposure in air is the…

a. Rad

b. Rem

c. Roentgen

2. With the use of a tissue weighting factor, the Roentgen is converted to a _______ for assessing absorbed dose.

a. Rad

b. Rem

c. Roentgen

3. The REM measures

a. Exposure in air

b. Air kerma

c. Relative Biological Effectiveness

4. The SI unit for the RAD is…

a. Sievert

b. Gray

c. LET

5. The SI unit for REM is …

a. Sievert

b. Gray

c. LET

6. What is the conversion from Rad / Rem to Sievert / Gray?

a. Multiply Rad / Rem by 100

b. Divide Rad / Rem by 100

c. Multiply by 0.96 for muscle

Page 54: Client Login - EXAM 1 Questions and Answers · 2015-09-29 · 18. Which of the following most affects the technetium yield from a 99Mo‐99mTc generator? a. Channeling b. Radiolysis

7. The average background radiation exposure is….

a. 100-200mRem

b. 300-500mRem

c. 500- 1000mRem

8. The largest source of background radiation is from…

a. Radon

b. Cosmic ray

c. Terrestrial radiation

9. The non-occupational dose limit is ….

a. 10mRem/yr

b. 100mRem/yr

c. 5000mRem/yr

10. The whole body occupational dose limit is..

a. 100mRem/yr

b. 500mRem/yr

c. 5000mRem/yr

11. The occupational dose limit to an extremity is…

a. 5000mRem/yr

b. 15,000mRem/yr

c. 50,000mRem/yr

12. The pregnant worker dose limit is…..

a. 100mRem/gestation

b. 500mRem/gestation

c. 5000mRem/gestation

13. Personnel monitoring is required when…

a. The annual exposure is likely to exceed 500mRem

b. Personnel are handling radioactive material.

c. Both a & b

Page 55: Client Login - EXAM 1 Questions and Answers · 2015-09-29 · 18. Which of the following most affects the technetium yield from a 99Mo‐99mTc generator? a. Channeling b. Radiolysis

14. Common monitors in Nuclear Medicine / Cardiology labs are…..

a. Film Badges

b. TLD ring badges

c. OSL whole body badges

d. All of the above

15. ALARA stands for…

a. As low as reasonably achievable

b. American legal and radiation association

c. Allow lower radiation as acceptable limit

16. 100mR/hr measured at 1 meter would be _______ at 2 meters.

a. 10mR/hr

b. 25mR/hr

c. 50mR/hr

17. Assuming the HVL for Tc99m is 0.3mm in lead. How much lead would be needed to reduce the exposure from 10mR to 5mR.

a. 0.1mm

b. 0.3mm

c. 0.6mm

18. The ALARA I level for the whole body is….

a. 25mRem/quarter

b. 125mRem/quarter

c. 375mRem/quarter

19. The appropriate survey instrument in Nuclear Medicine is the ….

a. Ion chamber

b. Pen Dosimeter

c. Geiger Mueller Meter (GM)

20. When checking for removable contamination with a scaler and well, the correct size area to wipe is….

a. 1cmx1cm

b. 10cm x 10cm

c. 100cmx100cm

Page 56: Client Login - EXAM 1 Questions and Answers · 2015-09-29 · 18. Which of the following most affects the technetium yield from a 99Mo‐99mTc generator? a. Channeling b. Radiolysis

21. When dealing with a Tc99m spill, the threshold for major and minor spill as defined by NUREG 1556 volume 9 is……

a. 100mCi

b. 50mCi

c. 25mCi

d. 10mCi

22. If holding radioactive waste for decay in storage, it must be held at least……

a. 1 half life

b. 10 half lives

c. 5 half lives

23. When returning radioactive material as “excepted package, limited quantity”, the surface exposure must be….

a. <0.1mR/hr

b. <0.5mR/hr

c. <10mR/hr

24. Misadministrations must be reported to the DOH if...

a. The patient is a young female

b. The dose was not administered by an MD

c. Results in a dose to the patient >5Rem whole body